chapter 16 Metabolic Diseases

Nicole Glaser, MD Elka Jacobson-Dickman, MD Nathan Kuppermann, MD, MPH, FAAP Debra L. Weiner, MD, PhD, FAAP

Objectives Chapter Outline Describe the 1 epidemiology, Introduction clinical features, Diabetic and emergency Hypoglycemia in Children With Diabetes management of Hypoglycemia in Nondiabetic Children diabetes mellitus (DM), Adrenal Insufficiency/Congenital Adrenal Hyperplasia adrenal insufficiency, syndrome of inappropriate secretion Syndrome of Inappropriate Secretion of Antidiuretic Hormone of antidiuretic Intoxication hormone (SIADH), Metabolic Disease of the Newborn and Young Child: Inborn water intoxication, and Errors of Metabolism diabetes insipidus (DI). Describe the fluid and 2 problems associated with these conditions. Discuss the clinical 3 features and emergency management of metabolic diseases of the newborn and young child.

Copyright © 2012 by the American Academy of Pediatrics and the American College of Emergency Physicians A 10-year-old boy presents to the emergency department (ED) with altered mental status and a 1 2-week history of polyuria and weight loss. He is experiencing abdominal pain. On examination, he has a respiratory rate of 36/min, a heart rate of 150/min, a systolic blood pressure of 80 mm Hg, and a temperature of 37.9°C (100.2°F). Pulse oximetry oxygen saturation is 97% on room air. On examination, he appears lethargic, has dry mucous membranes, has normal abdominal examination results, and has a nonfocal neurologic examination results. Bedside rapid glucose measurement is above the upper limit of the glucose meter (>500 mg/dL).

1. What therapeutic actions must be taken immediately? 2. What laboratory tests would you order? CASE SCENARIO SCENARIO CASE 3. What is the most important potential complication?

Introduction The emergency management of endocrine and Diabetic ketoacidosis (DKA) is the most impor- metabolic disorders presents diagnostic and tant complication of type 1 DM (Figure 16.1). therapeutic challenges. Many of these disor- Diabetic ketoacidosis occurs when insulin con- ders, such as congenital adrenal hyperplasia centrations are low relative to the concentrations (CAH) and inborn errors of metabolism, occur of counterregulatory hormones, particularly relatively infrequently. Nonetheless, rapid and glucagon. This imbalance results in hypergly- appropriate intervention is necessary to prevent cemia along with the breakdown of fat to form complications. Awareness of key clinical and ketone bodies, eventually resulting in . laboratory findings that differentiate endocrine Diabetic ketoacidosis occurs in 25% to 40% of and metabolic disorders from other entities with children with new onset of type 1 diabetes melli- similar presenting features is essential. tus (DM).1–3 Repeat episodes of DKA in children 16-3

Copyright © 2012 by the American Academy of Pediatrics and the American College of Emergency Physicians with known diabetes can occur during episodes causes of . Acidosis results in com- of illness or with diabetes mismanagement or pensatory tachypnea, and a characteristic fruity malfunction of diabetes care equipment (insulin (acetone) breath odor can often be detected. pumps). Presumed omission or incorrect ad- Because the hyperosmolar state allows for the ministration of insulin injections is the most relative preservation of intravascular volume, common cause of DKA in children with known some signs of dehydration can be less obvious DM (69%); bacterial infections are infrequently than in dehydrated patients with more normal present (13%).4 serum osmolality. Alterations in mental status can range from disorientation to depressed con- Clinical Features sciousness. This alteration of mental status can Diabetic ketoacidosis is characterized by hyper- glycemia and elevated serum ketone concentra- tions, resulting in acidosis. Inadequate insulin YOUR FIRST CLUE concentrations allow the partial oxidation of fatty acids to form ketone bodies. This process Signs and Symptoms of DKA then results in acidosis with an increased anion • History of polyuria, , weight gap. Hyperglycemia results in osmotic diuresis loss, nausea, and vomiting with polyuria, polydipsia, and eventual dehy- • Signs of dehydration dration. Presenting features of DKA include • Breath with fruity odor a history of polyuria, polydipsia, nausea, and vomiting. Abdominal pain and intestinal ileus • Abdominal pain are common features and can mimic an acute • Tachypnea abdomen. In a patient with clinical signs of de- • Lethargy hydration, the presence of polyuria helps dif- ferentiate DKA from and other

Pathophysiology of Diabetic Ketoacidosis

fatty acid oxidation gluconeogenesis glycogenolysis peripheral glucose uptake and metabolism

Ketone Bodies Hyperglycemia

Acidosis osmotic diuresis

vomiting increased renal Na+ loss renal insensible Dehydration Ca, phos, fluid losses Mg loss poor tissue perfusion renal K+ loss increased lactate

Figure 16.1 Diabetic ketoacidosis.

Behrman RE, Kliegman RM. Nelson Essentials of Pediatrics. 4th ed. Philadelphia, PA: WB Saunders; 2002:185. Reprinted with permission.

16-4 Metabolic Diseases

Copyright © 2012 by the American Academy of Pediatrics and the American College of Emergency Physicians be due to acidosis and dehydration but can also are total body concentrations of sodium, phos- reflect more serious underlying intracranial dis- phate, calcium, and magnesium. Although total ease. Rarely, children with DKA present to the body sodium is depleted, the degree of deple- ED with clinically apparent ,5 as tion is less than that suggested by the measured well as cerebral infarctions or thromboses. serum sodium concentration, which is artifi- cially depressed due to hyperglycemia. Blood Complications urea nitrogen (BUN) concentrations are usually The most frequent complications of DKA treat- elevated due to prerenal azotemia. The white ment include and hypoglycemia. blood cell count is often elevated and frequently Hypocalcemia, hypophosphatemia, and hypo- left-shifted as a result of the ketoacidotic state 4 magnesemia can also occur but are much less alone. It is generally unnecessary to search for common. Of the more serious complications of a source of infection beyond performing a care- DKA, clinically apparent cerebral edema is the ful physical examination unless fever or other most frequent, occurring in approximately 1% specific signs of infection are present. of DKA episodes. Cerebral edema is the most important diabetes-related cause of mortality in Differential Diagnosis children with type 1 DM.6,7 The presenting symptoms and signs of DKA Although cerebral edema can occur before can initially be confused with gastroenteritis or hospital treatment of DKA, this complication other gastrointestinal disorders that present with more typically occurs several hours after initia- vomiting and abdominal pain. Infections such as tion of therapy. The clinical presentation of ce- pneumonia, meningitis, and urinary tract infec- rebral edema can take different forms. Typically, tions also can present with , patients with cerebral edema experience head- tachypnea, and altered mental status or can be a ache and exhibit alterations in mental status. cause of the development of DKA. The history of Obtundation, papilledema, pupillary dilation polyuria despite clinical dehydration, absence of or anisocoria, blood pressure variability, brady- diarrhea, and presence of tachypnea with fruity cardia, and apnea can also occur in severe cases. breath odor should differentiate DKA from these Deep venous thromboses can occur in chil- other entities even before laboratory tests are or- dren with DKA, particularly when central ve- dered. Diabetic ketoacidosis must also be differ- nous catheters are used.8,9 Other complications entiated from hyperglycemic hyperosmolar state of DKA are rare but include cerebral infarctions (HHS) (formerly known as hyperglycemic hyper- or thrombosis, acute renal tubular necrosis lead- osmolar nonketotic coma), which presents with ing to renal failure, pulmonary edema, pulmo- severe hyperglycemia (serum glucose concentra- nary embolus, arrhythmia, pancreatitis, and tion >600 mg/dL and often >1,000 mg/dL) and intestinal necrosis. hyperosmolality (serum osmolality >330 mOsm), profound dehydration, and a depressed level of Diagnostic Studies consciousness, but mild or no ketosis.10 Hyper- Diabetic ketoacidosis is defined as hyperglyce- glycemic hyperosmolar state can in fact represent mia (serum glucose concentration >200 mg/dL), acidosis (venous pH <7.25 or serum bicarbon- ate concentration <15 mEq/L), and the presence THE CLASSICS of ketones in the serum or urine.6,7 Electrolyte abnormalities also occur as a result of urinary losses and shifts due to acidosis. High serum Laboratory Findings in DKA concentrations of potassium are often found at • Hyperglycemia the time of presentation of DKA, but normal • Metabolic acidosis or low concentrations can occur if the ketoaci- • Ketonemia or ketonuria dotic state is prolonged. Intracellular potassium concentrations are characteristically depleted, as

Diabetic Ketoacidosis 16-5

Copyright © 2012 by the American Academy of Pediatrics and the American College of Emergency Physicians should not be compromised by theoretical con- WHAT ELSE? cerns associated with fluid administration. The average degree of dehydration in chil- dren with DKA is approximately 7%.12 The Differential Diagnosis of DKA typical child with DKA should receive an ini- • Gastroenteritis tial fluid bolus of 10 mL/kg of isotonic fluid (ie, • Abdominal surgical emergency isonatremic fluids such as 0.9% saline [normal • Toxic ingestion saline] or lactated Ringer’s solution) for 30 to 60 • Infections (pneumonia, meningitis, minutes. If there is evidence of poor perfusion urinary tract infections) or significant hypovolemia, 20 mL/kg can be • Hyperglycemic hyperosmolar state used as the initial bolus. Greater administra- tion of fluids is indicated if signs of shock are present. Isotonic fluid boluses can be repeated if necessary to restore perfusion. Once perfusion has been restored, the remaining fluid deficit plus maintenance fluids can be replaced dur- one extreme in a continuum of presentations of ing the next 36 to 48 hours using half normal DKA and a mixed presentation with features of (0.45% sodium chloride solution) to normal DKA and HHS can also occur. Although HHS is saline (0.9% sodium chloride solution).6,7 uncommon in children, recent reports suggest Hyperglycemia 10 that the frequency might be rising. Hypergly- After the initial fluid bolus(es), insulin should cemic hyperosmolar state occurs more frequently be administered via continuous intravenous in patients with type 2 rather than type 1 diabe- (IV) infusion at a dosage of 0.1 U/kg per hour. tes and in children with neurologic abnormali- Maximal suppression of ketogenesis occurs rap- ties that lead to abnormal thirst mechanisms or idly using a continuous insulin infusion at this limited access to liquids. The treatment approach dosage and an initial IV bolus or “loading dose” for HHS differs from DKA. Discussion of HHS of insulin is unnecessary. When serum glucose treatment is beyond the scope of this chapter but concentrations fall below 250 to 300 mg/dL 10 has been reviewed elsewhere. during treatment, glucose should be added to the IV fluids to ensure that hypoglycemia does Management not occur. The insulin infusion rate generally The fundamental treatment measures for DKA should not be decreased until ketoacidosis re- include replacement of fluid deficits, correction solves (pH >7.30 and bicarbonate >15). This of acidosis and hyperglycemia via insulin ad- will like take roughly 10 to 24 hours to achieve, ministration, replacement of depleted electro- so the insulin infusion must often continue on lytes, and monitoring for complications. the inpatient service. Fluid Replacement Acidosis Optimal fluid management of DKA has been The acidosis in DKA is due primarily to two a subject of great controversy, mainly due to factors: the production of ketone bodies due to theoretical concerns about the role of fluid ad- relative insulin deficiency and the accumula- ministration in contributing to cerebral edema. tion of lactate due to dehydration and poor tis- Available evidence, however, does not confirm sue perfusion. Insulin treatment promotes the a primary role for fluid administration in caus- metabolism of ketone bodies and halts further ing this complication.5,11 The most compelling ketone production, whereas the administration evidence suggests that children who present of IV fluids treats the dehydration. This thera-

with low Pco2 and high BUN concentration and peutic combination is generally sufficient for those treated with bicarbonate are at higher risk the correction of acidosis. Bicarbonate admin- for cerebral edema.5 Appropriate restoration of istration generally should be avoided because perfusion and hemodynamic stability, therefore, associations between bicarbonate therapy and

16-6 Metabolic Diseases

Copyright © 2012 by the American Academy of Pediatrics and the American College of Emergency Physicians KEY POINTS

DKA Management Guidelines IV fluids: - Administer initial 10- to 20-mL/kg bolus of normal saline. Fluid bolus can be repeated if necessary to restore perfusion. - Subsequent fluid administration should replace the remaining fluid deficit (typically approximately 70 mL/kg) plus maintenance fluids during 36 to 48 hours using half normal to normal saline. Normal saline can be used initially with a transition to half normal saline after several hours. Insulin: - Insulin should be administered after initial fluid bolus(es). - Insulin should be administered as a continuous infusion of 0.1 U/kg per hour. - An initial insulin bolus dose is not necessary. - Insulin should be administered at the above rate until ketoacidosis resolves (pH >7.30 and bicarbonate >15 mEq/L). To prevent hypoglycemia, glucose-containing IV fluids should be used after the plasma glucose concentration declines below approximately 250 to 300 mg/dL (13.9 to 16.7 mmol/L). Electrolyte replacement: - For DKA patients presenting initially with hypokalemia, potassium replacement should begin immediately. If potassium levels are normal, potassium replacement should begin concurrent with insulin administration, provided that renal function is adequate. For patients with hyperkalemia, potassium replacement should begin when serum potassium levels decline to the normal range. - Potassium replacement should be given as potassium chloride (KCl) or a 50:50 mixture of potassium chloride (KCl) and potassium phosphate or potassium acetate to sum to 40 mEq of potassium per liter of IV fluids. Serum potassium concentrations should be monitored frequently and rates of administration adjusted according to measured concentrations. - Calcium, magnesium, and phosphate concentrations should be monitored. Phosphate concentrations typically decrease during treatment, and replacement of phosphate (see above) can be considered. Replacement of calcium and magnesium is rarely required. Bicarbonate treatment: - Bicarbonate treatment should be avoided except in rare circumstances of symptomatic hyperkalemia and/or cardiovascular instability caused by severe acidosis and not responding to fluid and insulin therapy. Monitoring: - Vital signs should be measured hourly. - Continuous cardiac monitoring is recommended. - Accurate recording of fluid intake and output should be performed. - Blood glucose concentrations should be measured hourly. , serum urea nitrogen,

creatinine, venous pH, and Pco2 should be measured every 2 to 4 hours. Calcium, magnesium, and phosphate should be monitored approximately every 6 hours. More frequent measurements might be needed in patients with abnormal or rapidly changing electrolyte concentrations. - Mental status should be assessed hourly or more frequently in patients with headache or other symptoms or signs of cerebral edema (see text). Admission to the pediatric intensive care unit vs pediatric ward: - Admission to the pediatric intensive care unit is recommended for children requiring more intensive monitoring or those at risk for complications. These patients include: - Children younger than 5 years - Children with altered mental status

- Children with very high initial BUN concentration and/or very low initial Pco2 levels or severe acidosis - Children with mixed presentation of DKA and hyperglycemic hyperosmolar state

Diabetic Ketoacidosis 16-7

Copyright © 2012 by the American Academy of Pediatrics and the American College of Emergency Physicians cerebral edema have been found in a large study ness of cerebral edema treatment interventions is of DKA5 and administration of bicarbonate has unclear, patients who have deterioration in mental not been found to hasten resolution of DKA.13 status generally should be treated for presumed In rare cases of symptomatic hyperkalemia or cerebral edema and subsequently evaluated us- acidosis severe enough to cause hemodynamic ing computed tomography or magnetic reso- instability not responsive to other therapeutic nance imaging (MRI). It is commonly believed measures, however, cautious administration of that early treatment with hyperosmolar agents, bicarbonate should be considered. such as mannitol or 3% saline (hypertonic saline), Electrolyte Imbalances in patients with DKA and cerebral edema can be 16 Regardless of the serum potassium concentration beneficial, although clinical data are lacking. The at presentation, total body potassium is depleted potential adverse effects of hyperosmolar agents in in patients with DKA. With the treatment of DKA, this setting, however, are relatively minor in rela- serum potassium concentrations can decrease tion to the potential for cerebral injury caused by rapidly as insulin and fluid therapy improve the cerebral edema, and therefore hyperosmolar treat- state of acidosis and potassium shifts to the in- ment should be strongly considered if cerebral tracellular space. Potassium should typically be edema is suspected. Aggressive hyperventilation added to the IV fluids at the time of initiation of has been shown to be associated with a greater insulin treatment, provided that adequate renal likelihood of an adverse outcome in DKA-related function is verified. For patients presenting with cerebral edema; therefore, this should probably hypokalemia, potassium replacement should be avoided unless necessary to avoid cerebral 16 begin sooner. For patients with initial hyper- herniation. kalemia, potassium replacement should begin after potassium levels decrease to the normal Hypoglycemia in Children With range. Potassium replacement should be given Diabetes as a mixture of potassium chloride and potas- Hypoglycemia, the most common complication sium phosphate or potassium acetate at a con- of type 1 diabetes in childhood, can occur in any centration of 30 to 40 mEq/L. Although severe child in whom the dose of administered insulin hypophosphatemia occurs rarely in children with exceeds insulin requirements. Albeit less frequent- DKA, the consequences (eg, rhabdomyolysis and ly, hypoglycemia can also occur in patients with 14,15 hemolytic anemia) can be devastating. There- type 2 diabetes on oral hypoglycemic or insulin fore, it seems prudent to administer a portion of regimens. Responses to hypoglycemia include the potassium replacement as the phosphate salt, release of counterregulatory hormones, such as although a thorough study of this issue is lacking. glucagon, epinephrine (adrenaline), cortisol, and Monitoring and Treating Complications growth hormone. However, over time both the Glucose concentrations should be measured glucagon and epinephrine (adrenaline) responses hourly using a bedside glucose meter throughout might be impaired, increasing the risk of persistent the period of IV insulin treatment. A reasonable hypoglycemia due to lack of adrenergic symptoms. rate of decline in the serum glucose concentration The American Diabetes Association criteria is approximately 50 to 100 mg/dL per hour.6,7 for the clinical classification of hypoglycemia With the initial infusion of IV fluids and restora- in patients with DM include all episodes of an tion of renal perfusion, however, glucose concen- abnormally low plasma glucose concentration trations might decrease more rapidly. Electrolyte (<70 mg/dL or <3.9 mmol/L) with or with- concentrations should be measured every 3 to 4 out symptoms that expose the individual to hours or more frequently if severe abnormalities harm.17,18 A glucose concentration of 70 mg/dL are present. is higher than the value used to diagnose hypo- Careful monitoring is essential for early detec- glycemia in people without diabetes; however, it tion of complications of DKA, particularly cerebral is considered an appropriate threshold because edema. Therefore, neurologic and mental status it approximates the lower limit of the physi- should be checked hourly. Although the effective- ologic fasting nondiabetic range, the normal

16-8 Metabolic Diseases

Copyright © 2012 by the American Academy of Pediatrics and the American College of Emergency Physicians glycemic threshold for glucose counterregu- cally impaired to a level that requires help from latory hormone secretion, and the highest a second person to administer oral therapy. In antecedent glucose level reported to reduce the absence of a person to aid therapy, moderate sympathoadrenal responses to subsequent hy- episodes could escalate in severity. Severe hypo- poglycemia.19 It has been debated that a cutoff glycemia is recognized by the presence of criti- value of less than 63 mg/dL (3.5 mmol/L) is cal neurologic symptoms (seizures or coma) or preferable to avoid overclassification of hypo- requirement for intervention with subcutaneous glycemia in asymptomatic patients.20–23 glucagon and/or IV dextrose. Case reports exist of In children, the risk of hypoglycemia is as- acute and transient cortical blindness and stroke- sociated with younger age,24 tighter glycemic like hemiparesis associated with severe hypogly- control,25 insulin regimens with fixed dosing (as cemia.32,33 Unlike mild episodes of hypoglycemia opposed to therapy that delivers a basal insulin that are relatively frequent in children with type dose/infusion rate with intermittent boluses to 1 DM, severe hypoglycemic events are uncom- cover for intake),26–29 exercise (due to increased mon, occurring at a rate of approximately 5 to insulin absorption, more rapid glucose utiliza- 19 events per 100 patient-years.1,2 tion, and increased insulin sensitivity), acute illness (particularly when associated with nau- Diagnostic Studies and Management sea, vomiting, and anorexia), psychological Patients with mild and moderate hypoglycemia disturbance, lower socioeconomic status, and should be treated orally with a concentrated and occurrence of one or more prior hypoglycemic rapidly absorbed simple carbohydrate source, 30 episodes. In children and adolescents receiv- such as glucose tablets, fruit juice, or cake frost- ing intensive therapy, the risk of severe hypogly- ing; 15 to 20 g of oral glucose is typically suf- cemia is increased more than threefold. Finally, ficient. A snack containing a mixed source the risk is also increased at night due to a sleep- should follow to sustain normal blood glucose induced blunted counterregulatory hormone level. Blood glucose levels should be checked 17,19,31 response to hypoglycemia. every 15 to 20 minutes for approximately 2 hours to confirm that glucose values have nor- Clinical Features malized and to determine whether further inter- Hypoglycemic symptoms can be adrenergic due vention is necessary. The patient might require to epinephrine (adrenaline) release and/or neuro- additional carbohydrates until normal blood glycopenic due to direct effects of hypoglycemia glucose levels are sustained. on the central nervous system (CNS). Adrenergic A patient with diabetes presenting with symptoms include tremor, pallor, tachycardia, altered mental status should have glucose palpitations, and diaphoresis. Neuroglycopenic concentration assessed rapidly with a bedside symptoms include fatigue, lethargy, headaches, glucose meter. Hypoglycemia, if present, should behavior changes, drowsiness, unconsciousness, be corrected as rapidly as possible with an IV seizures, or coma. The severity of the neuroglyco- infusion of 0.5 g/kg of dextrose. This can be penic symptoms increases with the severity of hy- accomplished by infusing 5 mL/kg of 10% dex- poglycemia and resultant CNS energy deprivation. trose in an infant, 2 mL/kg of 25% dextrose in The severity of hypoglycemia is classified a toddler, or 1 mL/kg of 50% dextrose in older by both symptoms and the clinical intervention children (note the “50 rule”: the product of the required.17 Mild hypoglycemia is associated with milliliters per kilogram dose and the percentage adrenergic and mild neuroglycopenic symptoms, of dextrose should equal 50). In the event that such as headaches and mood changes in older IV access cannot be established rapidly, gluca- children and poor feeding, lethargy, jitteriness, gon should be given via subcutaneous or intra- and hypotonia in infants and toddlers. In such muscular injection. The accepted dose is 0.02 cases, oral intake of a rapidly absorbed carbohy- to 0.03 mg/kg or 0.5 mg for children weighing drate is generally adequate treatment. Patients less than 20 kg and 1 mg for children weighing with moderate hypoglycemia are neurologi- greater than 20 kg. If neurologic or mental status

Hypoglycemia in Children With Diabetes 16-9

Copyright © 2012 by the American Academy of Pediatrics and the American College of Emergency Physicians abnormalities persist for prolonged periods after venous sample should be sent to the laboratory correction of hypoglycemia, other possibilities for measurement with greater accuracy. Treat- should be considered, such as toxic ingestions ment for hypoglycemia should not be delayed and other intracranial disease. while awaiting results. At the time that the confirmatory sample is drawn, an additional serum sample (enough for special studies) Hypoglycemia in Nondiabetic should be obtained for future measurement of Children hormone concentrations and other biochemi- cal measures. It is essential that this sample be Hypoglycemia can result from excessive insulin drawn at the time of the hypoglycemic event. production (insulinoma or hyperinsulinism) or Without information from this sample, it is usu- decreased concentrations of counterregulatory ally impossible to determine the cause of the hormones (deficiencies of growth hormone, hypoglycemic episode. If hypoglycemia is con- cortisol, or both). Hypoglycemia can also result firmed, the measurements to be obtained on the from inborn metabolic errors that impair gluco- initial sample include serum insulin, cortisol, neogenesis, glycogenolysis, or fatty acid oxida- and growth hormone concentrations, as well as tion. Unsuspected hypoglycemia can be found serum ketones, lactate, and free fatty acid when patients present to the ED with episodes concentrations.36 A toxicologic evaluation of acute illness. A recent study revealed that a for ethanol, salicylates, -blockers, and oral substantial percentage (28%) of these patients hypoglycemic agents should also be considered. had previously undiagnosed fatty acid oxidation Remaining serum from the initial sample should 34 defects (19%) or endocrine disorders (9%). be stored for additional biochemical testing if necessary. Finally, a urine sample should also Clinical Features be obtained and tested for ketones and reduc- Infants with hypoglycemia can present with ing substances. An additional sample of urine jitteriness, poor feeding, lethargy, hypotonia, should likewise be stored for future testing. hypothermia, apneic episodes, or seizures. Symptoms of hypoglycemia in infants might Differential Diagnosis also be subtle and less obvious than those in There are many possible causes of hypoglyce- older children. In older children, symptoms mia, and a discussion of all causes is beyond of hypoglycemia include headaches; vision the scope of this chapter. Among the more fre- changes; mental status changes, such as con- quent causes (in patients without DM) are toxic fusion, lethargy, irritability, or anxiety; and seizures. Adrenergic symptoms, such as palpita- tions, sweating, pallor, and tremulousness, are usually also present. Hypoglycemia should be WHAT ELSE? considered in all patients presenting with un- explained alterations in mental status, seizure, or loss of consciousness, and a routine bedside Differential Diagnosis of Hypoglycemia test for hypoglycemia should be part of pediatric • Toxins (oral hypoglycemic agents, resuscitation procedures.35 ᵝ-blockers, ethanol) • Hypopituitarism Diagnostic Studies • Adrenal insufficiency If hypoglycemia is suspected, a rapid glucose • Fatty acid oxidation defects test should be performed using a bedside blood • Hyperinsulinism glucose meter. A glucose concentration less than • Ketotic hypoglycemia 50 mg/dL should be considered abnormal and • Sepsis the patient treated. If the bedside test reveals a low glucose concentration, a confirmatory

16-10 Metabolic Diseases

Copyright © 2012 by the American Academy of Pediatrics and the American College of Emergency Physicians should equal 50). Subsequently, an IV infusion YOUR FIRST CLUE of 10% dextrose at 1.5 times maintenance rates should be provided to maintain glucose con- centrations in the normal range and reverse the Signs and Symptoms of Hypoglycemia catabolic state. If adrenal insufficiency is known • Infants: jitteriness, poor feeding, lethargy, hypotonia, hypothermia, apnea, seizures or strongly suspected and adequate samples have been set aside for further study, IV hydro- • Children: headaches, altered mental status, diaphoresis, pallor, palpitations, cortisone should be given (50 mg for children tremulousness, seizure younger than 4 years and 100 mg for children older than 4 years). ingestions (eg, oral hypoglycemic agents, Adrenal Insufficiency/ ᵝ-blockers, and ethanol), sepsis, hypopituita- Congenital Adrenal rism, adrenal insufficiency, fatty acid oxidation Hyperplasia defects, hyperinsulinism, and ketotic hypogly- cemia. Analysis of the initial serum sample will Adrenal insufficiency can be related to processes differentiate among many of these entities and directly affecting the adrenal gland (primary), or will indicate whether additional analyses are it can result from adrenocorticotropic hormone necessary to investigate less frequent causes of (ACTH) deficiency (secondary). Both primary hypoglycemia. and secondary adrenal insufficiency are char- acterized by inadequate production of cortico- Management steroids that can culminate in “adrenal crisis,” Hypoglycemia should be corrected as rapidly a life-threatening cardiovascular collapse. How- as possible with an IV infusion of 0.5 g/kg of ever, in secondary adrenal insufficiency miner- glucose (dextrose). This can be accomplished by alocorticoid function is preserved. infusing 5 mL/kg of 10% dextrose in an infant, The causes of adrenal insufficiency in child- 2 mL/kg of 25% dextrose in a toddler, and 1 hood are varied. Congenital adrenal hyperplasia mL/kg of 50% dextrose in older children (note is the most common form of primary adrenal the “50 rule”: the product of the milliliters per insufficiency in children, with an incidence of kilogram dose and the percentage of dextrose 1 in 10,000 to 18,000 live births.37 Congeni- tal adrenal hyperplasia is a group of autosomal recessive inherited adrenal steroidogenesis en- KEY POINTS zyme deficiencies (most often 21-hydroxylase) that cause various degrees of adrenal cortical inability to synthesize cortisol. In childhood, Diagnosis and Management of acquired primary adrenal insufficiency is rela- Hypoglycemia tively uncommon and can result from autoim- • Perform rapid bedside glucose testing mune adrenal destruction, infectious infiltration for altered level of consciousness, (by tuberculosis, most commonly worldwide), seizure, or lethargy. or inherited adrenoleukodystrophy syndromes. • Obtain additional blood and urine samples at the time of the hypoglycemic Furthermore, long-term treatment with glu- event. cocorticoids can result in secondary adrenal • Administer dextrose: 5 mL/kg of insufficiency by the suppressive effect on the 38 10% dextrose in an infant, 2 mL/kg hypothalamic-pituitary-adrenal axis. In fact, of 25% dextrose in a toddler, and the most common cause of acute adrenal insuffi- 1 mL/kg of 50% dextrose in older children. ciency in North America today is glucocorticoid withdrawal or omission in patients being treated for a variety of disorders with pharmacologic

Adrenal Insufficiency/Congenital Adrenal Hyperplasia 16-11

Copyright © 2012 by the American Academy of Pediatrics and the American College of Emergency Physicians corticosteroid doses administered by various routes. Treatment courses as brief as 2 weeks can result in transient suppression of endogenous cortisol production.39

Clinical Features Clinical manifestations of adrenal insufficiency depend on the age of the patient, the cause, and whether it is a chronic or an acute process. All cas- es must be considered potentially life-threatening. Infants with adrenal insufficiency can pres- Figure 16.2 Midline facial malformation. ent with poor feeding, vomiting, and lethargy. On physical examination, patients are typically hypotensive and have tachycardia. In primary secondary adrenal insufficiency is commonly as- adrenal insufficiency, electrolyte abnormalities sociated with signs and symptoms of additional can occur due to additional mineralocorticoid pituitary hormone deficiencies, such as growth deficiency. Severe hyperkalemia can cause life- failure, delayed puberty, secondary hypothy- threatening cardiac arrhythmias. Depending on the specific enzymatic defect and karyotype of roidism, and/or diabetes insipidus (DI). an infant with CAH, the genitalia can appear Adrenal crisis continues to occur in children malformed. In the most common form of CAH with known primary or secondary adrenal in- (21-hydroxylase deficiency), genetic female sufficiency during intercurrent illness because infants will likely have ambiguous genitalia, of failure to increase glucocorticoid dosing. whereas genetic male infants have normally It is important to recognize that other formed genitalia. Infants of both sexes with hormones affect cortisol metabolism and can CAH can have skin hyperpigmentation, particu- influence the onset or progression of adrenal larly in the genitalia, due to high concentrations insufficiency. It is well documented that ini- of ACTH. The salt-wasting form of classic CAH tiation of thyroid hormone replacement in an presents with a primary metabolic crisis in the individual with hypothyroidism accompanied by first weeks of life, regardless of sex. Infants with unrecognized adrenal insufficiency can precipi- secondary adrenal insufficiency due to hypopi- tate an adrenal crisis. The mechanism that pre- tuitarism might have midline facial malforma- cipitates adrenal crisis is not fully understood, tions (Figure 16.2) and male infants might have but it is hypothesized that hypothyroid patients a microphallus. have reduced cortisol requirements secondary Patients of all ages with acute adrenal insuf- to a reduced metabolic rate.41,42 When thyroxine ficiency can present with dehydration, hypoten- therapy is initiated, the metabolic rate and cortisol sion, hypoglycemia, or altered mental status. requirements increase, and an adrenal crisis can Hypoglycemia is more common in young chil- ensue. Accordingly, hyperthyroidism can increase dren. Altered mental status can occur at any age cortisol metabolism. It is suggested that in the with or without hypoglycemia.40 Patients with individual with hyperthyroidism and adrenal chronic adrenal insufficiency usually experience insufficiency, cortisol replacement should be in- fatigue, anorexia, nausea, vomiting, loss of ap- creased as much as twofold because of increased petite, weight loss, and recurring abdominal cortisol clearance.38 Growth hormone appears to pain. Salt craving is common in chronic primary decrease conversion of inactive cortisone to ac- adrenal insufficiency, as is postural hypotension. tive cortisol. Therefore, after growth hormone Hyperpigmentation and salt craving are not ob- therapy is initiated, a patient with unrecognized served in patients with secondary adrenal insuf- concomitant adrenal dysfunction might be vul- ficiency. Furthermore, unless there is a history nerable.43 Pregnancy is associated with increased of recent pharmacologic glucocorticoid therapy, corticosteroid-binding globulin and therefore

16-12 Metabolic Diseases

Copyright © 2012 by the American Academy of Pediatrics and the American College of Emergency Physicians lower free cortisol levels during the last trimester, thus necessitating an increase in hydrocortisone YOUR FIRST CLUE dosing by 50%. In addition, increasing intrapar- tum progesterone levels antagonize the mineralo- Signs and Symptoms of Adrenal corticoid effect, which can dictate adjustment of Insufficiency fludrocortisone supplementation as well.38 • Infants: poor feeding, inadequate weight Certain medications interfere with cortisol gain, vomiting, lethargy, hypotension, metabolism as well and must be considered in and tachycardia, with or without patients with adrenal disorders. Some drugs hyperpigmentation or masculinization of inhibit cortisol biosynthesis, including amino- genetic female genitalia glutethimide, etomidate, ketoconazole, metyra- • Children: fatigue, weight loss, and pone, medroxyprogesterone, and megestrol.39,44 postural hypotension, with or without In addition, certain drugs accelerate cortisol hyperpigmentation, and abdominal pain metabolism, such as phenytoin, barbiturates, • Other clues: history of CNS surgery 40 or tumors or prolonged or long-term and rifampin. corticosteroid use Diagnostic Studies Blood should be drawn to test for cortisol, elec- disorders, neurologic diseases, or child abuse. trolytes, glucose, ACTH, plasma renin activity, The presence of hyperkalemia, , and aldosterone before glucocorticoid therapy, acidosis, skin hyperpigmentation, and mascu- if possible. When CAH is suspected, serum linized genitalia in a genetic female can help 17-hydroxyprogesterone concentration should to differentiate primary adrenal insufficiency be measured as well. Measurement of urinary from these other entities. In an older child, sodium and potassium concentrations is helpful mild symptoms of primary adrenal insufficien- in assessing mineralocorticoid status and in dif- cy might be confused with mononucleosis or ferentiating from other hyponatremic conditions. anorexia nervosa. More severe symptoms can In primary adrenal insufficiency, testing mimic infections (eg, sepsis), hematologic dis- typically reveals hyperkalemia, hyponatremia, ease, psychiatric illness, or even an acute abdo- hypoglycemia, and acidosis. However, not all men. Again, the presence of hyperpigmentation patients manifest all of these laboratory abnor- and the characteristic laboratory abnormalities malities, and the diagnosis cannot be excluded serve to differentiate these entities. Secondary based on the absence of one or more of these adrenal insufficiency can be even more diffi- findings. Both BUN and creatinine concentra- cult to differentiate from other entities because tions can be elevated due to dehydration. In sec- of the lack of hyperpigmentation and the lack ondary adrenal insufficiency, hyperkalemia and of specific laboratory features other than hy- hyponatremia are absent due to preservation of poglycemia. In these cases, a history of CNS mineralocorticoid production, and hypoglycemia might be the primary biochemical abnormality.

Differential Diagnosis THE CLASSICS Most of the symptoms of adrenal insufficiency are nonspecific, and many overlap with other, Classic Laboratory Findings in Adrenal more common, diagnoses. In infancy, for ex- Insufficiency ample, adrenal insufficiency might be confused • Primary adrenal insufficiency: with other illnesses that present with lethargy, hyperkalemia, hyponatremia, vomiting, poor weight gain, hypotension, and hypoglycemia, and acidosis tachycardia, including sepsis, gastrointestinal • Secondary adrenal insufficiency: hypoglycemia malformations, congenital heart disease, in- born errors of metabolism, certain hematologic

Adrenal Insufficiency/Congenital Adrenal Hyperplasia 16-13

Copyright © 2012 by the American Academy of Pediatrics and the American College of Emergency Physicians tumors, neurosurgical procedures, congenital CNS malformations, other coincident pituitary WHAT ELSE? hormone deficiencies (suggested clinically by short stature, inappropriately absent or delayed puberty, or polyuria, as examples), or prolonged Differential Diagnosis of Adrenal Insufficiency exogenous glucocorticoid use must be relied on • Infants: infections, gastrointestinal for the diagnosis. Secondary adrenal insufficien- malformations, congenital heart disease, cy can evolve over time in patients with cranial inborn errors of metabolism, hematologic radiation, septo-optic dysplasia, autoimmune disorders, neurologic disease, or child hypophysitis, and head trauma.45–47 abuse • Children: mononucleosis, anorexia Management nervosa, infections, hematologic disease, psychiatric illness, or an acute abdomen In the hypotensive patient with adrenal insuf- ficiency, it is imperative to rapidly restore intra- vascular volume with isotonic sodium chloride containing glucose and to administer glucocor- ticoids expeditiously. An initial fluid bolus of is even suspected. If the patient has good gas- normal saline (20 mL/kg) should be given and trointestinal function, fludrocortisone (0.1 to repeated as necessary to improve perfusion. Af- 0.2 mg/d), a synthetic mineralocorticoid, can be ter the initial bolus(es), a continuous infusion of administered orally. However, typically, admin- normal saline with 5% dextrose should be given. istration of IV sodium chloride along with stress On the basis of body surface area, the recom- doses of hydrocortisone are sufficient to begin mended stress dose of IV hydrocortisone is 50 to normalizing electrolyte abnormalities, making 75 mg/m2 initially followed by 50 to 75 mg/m2 the addition of mineralocorticoid unnecessary per day divided into four doses.40 It is reason- in the initial phase of treatment.48 able to approximate IV hydrocortisone dosing Severe hyperkalemia, resulting in a non- and administer immediately 50 mg for children perfusing dysrhythmia, should be treated im- younger than 4 years and 100 mg for children mediately with IV calcium gluconate to restore older than 4 years. The benefits of sufficient a perfusing rhythm. This should be followed by dosing greatly outweigh the risk of underdos- IV sodium bicarbonate, nebulized albuterol (sal- ing in such cases. Hydrocortisone can be given butamol), and/or glucose with insulin to shift intramuscularly if no IV access exists, although the effects of intramuscular administration is slower and can be ineffectively absorbed if pe- KEY POINTS ripheral perfusion is poor. Comparable body surface area stress doses are 10 to 15 mg/m2 for methylprednisolone and 1.5 to 2 mg/m2 for Management of Adrenal Insufficiency dexamethasone; the latter two corticosteroids • Initiate fluid resuscitation (20 mL/kg of have minimal mineralocorticoid activity. Pred- normal saline bolus). nisone is not a preferred glucocorticoid because • Administer hydrocortisone (50 mg for child <4 years and 100 mg for child >4 it must be converted to prednisolone for it to years). have glucocorticoid activity, and in patients with • Treat symptomatic hyperkalemia (calcium liver failure, this conversion might be impaired. gluconate, bicarbonate, insulin, albuterol Dexamethasone can be used if one desires to [salbutamol]). treat the patient urgently but wishes to perform • Treatment consists of stress dosages of a diagnostic ACTH-stimulation test because this hydrocortisone, IV fluids, and correction glucocorticoid will not interfere with diagnostic of hyperkalemia. laboratory assays. Treatment should never be withheld if the diagnosis of adrenal insufficiency

16-14 Metabolic Diseases

Copyright © 2012 by the American Academy of Pediatrics and the American College of Emergency Physicians potassium intracellularly. If there is a coexisting gene or autosomal recessive or dominant muta- cardiomyopathy and/or acute renal failure pro- tions in the AQP-2 gene. These cases typically hibiting rapid rehydration, aggressive therapy present in infancy.56 Acquired nephrogenic DI of hyperkalemia might be needed. can be caused by various conditions, includ- ing primary renal disease, obstructive uropathy, hypokalemia, hypercalcemia, sickle cell disease, Diabetes Insipidus and medications such as lithium and demeclo- Under normal circumstances, plasma osmolality cycline (a tetracycline). In addition, prolonged is maintained within a narrow range (280–295 polyuria of any cause can also lead to some de- mOsm/L). This homeostasis requires adequate gree of nephrogenic DI because of a reduction of water intake, regulated by an intact thirst mech- tonicity in the renal medullary interstitium and anism and appropriate free water excretion by a subsequent decrease in the gradient necessary 50,53,57 the kidneys, which is mediated by adequate to concentrate urine. posterior pituitary secretion and peripheral ac- tion of arginine (AVP), also known Clinical Features as antidiuretic hormone (ADH). Diabetes insipi- Characteristic symptoms of DI are pronounced dus is caused by a deficiency in the produc- polyuria and polydipsia. Older children typi- tion or secretion of ADH (central DI) or due to cally present without any obvious abnormalities resistance to ADH in the kidneys (nephrogenic on physical examination as long as they have an DI).49,50 Symptoms of DI result from the effec- intact sense of thirst, access to fluids, and no tive inability to reabsorb free water at the level ongoing losses, such as diarrhea. They might of the collecting duct in the nephrons of the report a history of nocturia and secondary en- kidney. Polyuria, polydipsia, and hypo-osmolar uresis. Infants, in addition to polyuria and poly- urine characterize this disorder. dipsia, frequently demonstrate irritability, poor might be present as well at the time of diagnosis, weight gain, failure to thrive, hyperthermia, and particularly in infants or children with develop- signs of dehydration. Developmental delays mental delay. An intact thirst mechanism and and arrest of variable severity can ensue after access to water minimize the risk of repeated episodes of dehydration if the condi- dehydration and hypernatremia. tion remains untreated. Hydronephrosis can be Central DI is rarely congenital and more detected in all age groups. Interestingly, symp- frequently acquired. Congenital central DI can toms of DI might not be apparent in patients be caused by structural malformations or by with coexisting untreated anterior pituitary- autosomal dominant or recessive mutations in mediated adrenal glucocorticoid insufficiency the gene encoding vasopressin-neurophysin II.51 because cortisol is required to generate normal Acquired forms of central DI can occur at any age free water excretion.58 in association with a variety of disorders in which there is destruction or degeneration of vaso- Diagnostic Studies pressinergic neurons that can be caused by CNS Infants generally present with hypernatremia tumors (craniopharyngioma or germinoma), and serum hyperosmolality because they have infections (meningtis or encephalitis), head trauma, limited access to fluids. Coincident inappro- neurosurgery, vascular disorders, granuloma, priately dilute urine (<300 mOsm/L) helps to and autoimmune disorders (lymphocytic in- establish the diagnosis. Older children usually fundibuloneurohypophysitis). Idiopathic DI is do not present with hypernatremia or hyperos- a diagnosis of exclusion and one that is made molality unless their fluid intake is restricted, with decreasing frequency as a result of improved their thirst mechanism is abnormal, or they are sensitivity of diagnostic technology.50,52–55 experiencing excessive losses. In unclear cases, Much like central DI, nephrogenic DI can a carefully monitored “water deprivation test” be genetic or acquired. The genetic causes de- is recommended, whereby the fasting patient scribed are inactivating mutations of the AVPR2 is monitored for 8 to 10 hours with serial

Diabetes Insipidus 16-15

Copyright © 2012 by the American Academy of Pediatrics and the American College of Emergency Physicians YOUR FIRST CLUE THE CLASSICS

Signs and Symptoms of DI Laboratory Findings in Infants and • Infants: failure to thrive, irritability, Children With DI hyperthermia, dehydration • Infants: hypernatremia, hyperosmolality, • Children: polyuria, polydipsia, absence inappropriately dilute urine of DM • Children: dilute urine, nocturia, secondary enuresis

measurements of body weight, serum sodium and osmolality, and urine volume and osmolality. Differential Diagnosis If urine osmolality of greater than 750 mOsm/L The principal presenting sign of DI is polyuria, is achieved with any degree of water depriva- which, in addition to deficiency or impaired tion, DI can be excluded.59 The diagnosis of DI responsiveness to ADH, can result from an os- is established at any time point if serum motic agent, such as hyperglycemia as seen in osmolality rises above 300 mOsm/L and DM, or from excessive water intake (primary urine osmolality remains below 300 mOsm/L polydipsia). Patients with hypercalcemia or Urine osmolality in the 300- to 750-mOsm/L hypokalemia can also have impaired urinary range during water deprivation can indicate par- concentrating ability, resulting in polyuria. tial DI.53 During the test, if DI is suspected, an ADH plasma sample should be obtained, and Management then ADH, or a synthetic analog (desmopres- sin), should be administered to further distin- Patients without hypernatremia and dehydration guish ADH deficiency (central DI) from ADH do not require immediate intervention for DI unresponsiveness (nephrogenic DI). but should undergo evaluation to determine the If central DI is suspected, an MRI of the underlying cause. Patients with hypernatremic brain, with particular attention to the hypo- dehydration and hyperosmolality should be thalamic-pituitary region, is indicated. The treated with fluid resuscitation. This includes an posterior pituitary hyperintensity (bright spot) initial 20-mL/kg fluid infusion of normal saline on T1-weighted MRIs is often absent in central to restore perfusion followed by subsequent slow DI.54,60–62 However, the absence of the bright correction because overly aggressive free water spot is neither sensitive nor specific for this di- replacement can lead to cerebral edema. In ad- agnosis because it can be absent in healthy in- dition to correcting the fluid deficit, the manage- dividuals63 and, conversely, present in children ment of central DI includes treating the primary with central DI as well.64 In central DI patients, disease and normalization of urine output with a normal MRI does not exclude the possibility of an evolving occult hypothalamic-stalk lesion, and therefore follow-up with cerebrospinal flu- WHAT ELSE? id (CSF) tumor markers and cytology, serum tumor markers, and serial contrast-enhanced brain MRIs for early detection is critical.55 In- Differential Diagnosis of DI tracerebral calcifications of the frontal lobes and • Hyperglycemia from DM basal ganglia can result from repeated episodes • of dehydration.65 Ultrasonography can be help- • Hypercalcemia ful in detecting nonobstructive hydronephrosis, • Hypokalemia hydroureter, and megabladder, resulting from longstanding polyuria and polydipsia.66

16-16 Metabolic Diseases

Copyright © 2012 by the American Academy of Pediatrics and the American College of Emergency Physicians agents (eg, phenothiazines or tricyclic antidepres- THE BOTTOM LINE sants); (4) various pulmonary disorders and in- terventions (eg, pneumonia, asthma, or positive pressure ventilation); (5) non-CNS tumors with Key Concepts in the Diagnosis and Management of DI ectopic production of ADH; (6) patients treated with desmopressin acetate who drink excessive • DI should be suspected in children without DM who present with polyuria amounts of fluids (typically habit driven); and (6) and dilute urine. miscellaneous causes, such as AIDS, postopera- • Hypernatremia generally will not occur in tive state, glucocorticoid deficiency, and severe children with normal thirst mechanisms hypothyroidism.70,71 and free access to water. • Hypernatremia is more frequently Clinical Features encountered in infants. Patients with SIADH frequently will not mani- • ED management includes cautious fluid administration to correct dehydration fest symptoms of hyponatremia until the serum while promoting gradual hypernatremia sodium concentration falls below 120 mEq/L. correction and, in appropriate situation, Below this level, symptoms can include anorex- diagnostic evaluation initiation. ia, nausea, vomiting, lethargy, irritability, and confusion. Severe hyponatremia can culminate in seizures and loss of consciousness. desmopressin. Desmopressin is an ADH analog Diagnostic Studies with markedly reduced pressor activity in com- On the basis of criteria established by Bartter and parison with native ADH and a longer half-life Schwartz,72 the diagnosis of SIADH is made when and can be administered orally, intranasally, or the following constellation occurs: (1) plasma by subcutaneous injection. In infancy, if polyuria hypo-osmolality (<275 mOsm/L), (2) less than is not excessive, central DI can be best managed maximally dilute urine (urine osmolality >100 with extra fluid intake alone to avoid a potential mOsm/L), (3) clinical euvolemia, (4) natriuresis risk of hyponatremia associated with desmopres- (urine sodium >40 mEq/L), (5) normal renal func- sin treatment. In cases of nephrogenic DI, the tion, and (6) no evidence of thyroxine or corti- mainstay of treatment, particularly in infancy, sol deficiency. Typically, BUN concentrations are is thiazide in combination with either normal or low. Causes of pseudohyponatremia 65–68 amiloride or indomethacin. should be excluded such as that due to hyperlip- idemia, hyperproteinemia, and the osmotic effect of hyperglycemia. Most patients with SIADH have Syndrome of Inappropriate inappropriately measurable or elevated levels of Secretion of Antidiuretic plasma ADH relative to plasma osmolality. Hormone Differential Diagnosis Syndrome of inappropriate secretion of antidi- uretic hormone (ADH or AVP) is characterized by Other causes of hyponatremia include water in- excessive kidney resorption of free water, result- toxication (primary polydipsia or iatrogenic), ing in hyponatremia and hypo-osmolar serum hyponatremic dehydration, adrenal insufficien- in association with inappropriately concentrated cy, renal failure, medications, nephrotic urine and natriuresis.69–71 Several disorders and syndrome, cirrhosis, congestive heart failure, medications are associated with SIADH: (1) CNS severe hypothyroidism, cystic fibrosis, and disorders or damage, such as meningitis or en- cerebral salt-wasting (CSW) syndrome. These cephalitis, cerebral hemorrhage, head trauma, or diagnoses can generally be excluded based on after neurosurgical procedures; (2) psychiatric the absence of corresponding diagnostic signs, disorders; (3) a large variety of pharmacologic such as dehydration, edema, and hyperkalemia.

Syndrome of Inappropriate Secretion of Antidiuretic Hormone 16-17

Copyright © 2012 by the American Academy of Pediatrics and the American College of Emergency Physicians Management YOUR FIRST CLUE The mainstay of SIADH therapy for patients with mild or no symptoms is fluid restriction, Signs and Symptoms of SIADH treatment of the underlying disorder, or discon- tinuation of use of an offending drug. Ultimately, • Hyponatremia in association with head trauma, cerebral hemorrhage, or other replacement of sodium loss might also be nec- intracranial conditions essary, but it can usually be achieved through • Clinical symptoms and signs of dietary salt intake. Severe hyponatremia (serum hyponatremia, including anorexia, nausea, sodium <120 mEq/L), particularly when asso- vomiting, lethargy, irritability, altered level ciated with CNS disturbance (lethargy, coma, of consciousness, and seizures or seizures), might require treatment with IV hypertonic saline (3% sodium chloride solu- tion). The correct dosage of 3% sodium chlo- ride solution can be calculated based on the Euvolemia in chronic SIADH is a critical assumption that 12 mL/kg of 3% sodium chlo- distinguishing factor in the evaluation of a pa- ride solution will increase the serum sodium tient with serum hypo-osmolality; it is thought concentration by 10 mEq/L. It is generally rec- to represent an adaptation to water overload me- ommended that plasma sodium be corrected at diated, in part, at the cellular level through de- a rate of no greater than 0.5 mEq/L per hour, pletion of intracellular electrolytes (potassium) with an overall correction that does not exceed and organic osmolytes.70 Natriuresis, thought to 12 mEq/L in the initial 24 hours and 18 mEq/L be mediated in part through secretion of atrial in the initial 48 hours of treatment. Therapy natriuretic peptide, also contributes to volume should continue until neurologic symptoms regulation in chronic SIADH.73 improve and a safe level of approximately 120 The CSW syndrome is particularly important to 125 mEq/L is achieved. Further treatment to exclude because it occurs in overlapping clini- can then be accomplished with fluid restriction. cal settings with SIADH.74 Cerebral salt-wasting Serial rapid bedside serum sodium testing is syndrome is characterized by renal sodium and preferred when administering hypertonic saline fluid loss. However, the hypo-osmolality, hypo- natremia, and natriuresis in CSW syndrome are associated with volume contraction and clinical signs of dehydration (including elevated BUN and creatinine concentrations), helping to dis- WHAT ELSE? tinguish this syndrome from SIADH.70 Differential Diagnosis of SIADH • Water intoxication • CSW syndrome THE CLASSICS • Hyponatremic dehydration • Adrenal insufficiency • Renal failure Laboratory Findings in Patients With SIADH • Diuretic use • Serum hypo-osmolality (<275 mOsm/kg) • Nephrotic syndrome • Urine hyperosmolality (>100 mOsm/L) • Hypothyroidism • Inappropriate natriuresis (>40 mEq/L) • Cystic fibrosis • Normal renal, thyroid, and adrenal • Congestive heart failure cortical function • Cirrhosis

16-18 Metabolic Diseases

Copyright © 2012 by the American Academy of Pediatrics and the American College of Emergency Physicians well. Certain psychiatric illnesses, particularly KEY POINTS schizophrenia, can be associated with compul- sive water drinking. It is presumed that a central defect in thirst regulation plays an important Management of SIADH role in the pathogenesis of primary polydipsia • Restrict fluids in patients with mild or no in these patients.77–80 In some cases, for example, symptoms. the osmotic threshold for thirst is reduced below • Treat with 3% sodium chloride in cases of 81 severe hyponatremia (serum sodium <120 the threshold for the release of ADH ; therefore, mEq/L), particularly when associated with these patients will continue to drink until the neurologic disturbance. plasma tonicity is less than the threshold level. • Rapid correction of chronic hyponatremia Drug therapy can also contribute to the increase can precipitate central pontine in water intake in schizophrenic patients. Many myelinolysis. antipsychotic drugs induce the sensation of a dry mouth, which will enhance thirst.82

Clinical Features Clinical manifestations of water intoxication re- to avoid excessive sodium correction. If SIADH sult from hyponatremia. Infants typically present and hyponatremia are acute (<48 hours), it is with poor feeding, vomiting, and lethargy. Hypo- thought that hyponatremia can be corrected thermia and edema might also be present. With rapidly. However, if SIADH and hyponatremia severe hyponatremia, seizures can occur and are chronic (>48 hours), overzealous treatment might be prolonged. It has been suggested that can result in CNS damage, including central hypothermia can be a specific marker for hypo- pontine myelinolysis.70 Concurrent use of a di- natremia in infants with a new onset of seizures.83 uretic, such as furosemide, might be indicated if volume expansion is severe. Other therapeutic Diagnostic Studies approaches include the use of agents that in- duce nephrogenic DI, such as demeclocycline Hyponatremia (serum sodium concentration and lithium, although both are contraindicated, <130 mEq/L) with coinciding dilute urine is particularly in younger pediatric patients, be- the hallmark of this condition. Serum potas- cause of adverse effects. Urea has been used as sium, urea nitrogen, and creatinine concentra- an osmotic diuretic in pediatric SIADH.75 Once tions are typically normal, and acidosis is not patient stabilization is achieved, a careful search present, which helps exclude other causes of for an underlying cause for SIADH is necessary hyponatremia. Fictitious hyponatremia should if the cause is unknown. be excluded which can occur with hyperlipid- emia, hyperproteinemia, or due to the osmotic effect of hyperglycemia. Water Intoxication Water intoxication occurs when daily water Differential Diagnosis intake is excessive in relation to sodium. In The differential diagnosis of hyponatremia is ex- infants, this situation occurs most frequently tensive and includes renal failure, diuretic use, because of inappropriate dilution of formulas or renal tubular defects (), misguided supplemental feedings of solute-free SIADH, adrenal insufficiency, cystic fibrosis, ne- water.15,16 Excess water ingestion during infant phrotic syndrome, cirrhosis, and congestive heart swimming lessons has also been associated with failure. Iatrogenic hyponatremia can also occur this condition. Iatrogenic water intoxication is in infants or children when insensible losses are a well-known phenomenon, particularly in la- replaced with free water or other fluids deficient bor and delivery rooms.76 Water intoxication in sodium; this scenario differs, however, from can be seen as a manifestation of child abuse as classic water intoxication in that clinical and

Water Intoxication 16-19

Copyright © 2012 by the American Academy of Pediatrics and the American College of Emergency Physicians YOUR FIRST CLUE KEY POINTS

Signs and Symptoms of Water Diagnosis and Management of Water Intoxication Intoxication • Poor feeding • Presents with hyponatremia with a • Vomiting history of excessive water intake (relative to sodium). • Lethargy • Infant who presents with hypothermia • Seizures with seizures is a specific indicator for • Hypothermia hyponatremia. • Free water restriction and normalization of sodium intake are sufficient treatment in most instances. • Hypertonic 3% sodium chloride solution laboratory features of dehydration are present treatment can be used prudently in and the urine is concentrated. A careful history, severely affected patients. including infant feeding practices and psychiatric illness, in addition to the detection of dilute urine can lead to the diagnosis of water intoxication. THE CLASSICS Management For infants who are asymptomatic or have Laboratory Findings in Patients With mild symptoms of hyponatremia, reinstitution Water Intoxication of normal daily sodium with more restricted • Hyponatremia (sodium concentration fluid intake is generally all that is required. For <130 mEq/L) infants with severe manifestations of hyponatre- • Dilute urine mia (pronounced lethargy, seizures, or coma), • Absence of metabolic acidosis and treatment with 3% sodium chloride solution dehydration should be initiated with caution.

16-20 Metabolic Diseases

Copyright © 2012 by the American Academy of Pediatrics and the American College of Emergency Physicians A 14-month-old girl presents to the ED with a 1-day history of tactile fever, loss of appetite, and intermittent emesis. This morning, she did not seem interested 2 in her usual breakfast and seemed sleepier than usual. On examination, she has respirations of 40/min, a heart rate of 170/min, blood pressure of 80/40 mm Hg, and a temperature of 36.0°C (96.8°F). She appears lethargic and has mildly dry mucous membranes and no obvious source of infection. Bedside rapid glucose measurement reveals a blood glucose level of 20 mg/dL.

1. What therapeutic measures must be taken immediately? 2. What laboratory tests would you order? 3. What complications can occur as part of the disease process and secondary to CASE SCENARIO SCENARIO CASE therapeutic intervention?

Metabolic Disease of the peroxisomes), metals, or complex molecules.90 Newborn and Young Child: Most IEMs are due to a defect in an enzyme or transport protein that results in a block in a Inborn Errors of Metabolism metabolic pathway. Effects are caused by toxic Early recognition of a possible inborn error of accumulations of substrates before the block or metabolism (IEM) in acutely ill patients is critical from alternative metabolic pathway intermediates to minimize the high morbidity and mortality (disorders of protein metabolism, carbohydrate rates associated with these conditions. Although intolerance, metal metabolism, and porphyrias); individually rare, IEMs collectively are relatively defects in energy production and use within the common.84 A diagnosis of IEM should be con- brain, heart, liver, skeletal muscle, and other or- sidered not only in the evaluation and treatment gans due to a deficiency of products beyond the of critically ill patients but also in patients with block (disorders of carbohydrate metabolism [eg, recurrent episodes of vomiting, lethargy, and/or gluconeogenesis, glycogenolysis, and glycogen seizures and those with chronically progressive storage disorders], fatty acid oxidation defects, neurologic abnormalities and/or organ dysfunc- and mitochondrial disorders); or disturbances in tion. Usually, IEMs manifest in the neonatal pe- the synthesis or catabolism of complex molecules riod or infancy; however, presentation, can occur (lysosomal storage and peroxisomal disorders or at any time throughout childhood and even in defects in the metabolism of cholesterol, purines adulthood. Initial treatment of the critically ill and pyrimidines, and neurotransmitters; and de- child with an IEM does not require a detailed fects in glycosylation).85 Most IEMs have multiple knowledge of individual IEMs or biochemical forms that differ in age of clinical onset, severity, pathways but rather an understanding and early and even mode of inheritance. The IEMs most recognition of life-threatening clinical manifes- likely to result in acute, potentially life-threaten- tations and consequences of the metabolic de- ing decompensation include disorders of protein rangements.85–87 Prompt initiation of appropriate metabolism, defects in carbohydrate metabolism, treatment is crucial for optimizing immediate and fatty acid oxidation defects, and early-onset forms long-term outcome and is often lifesaving. of mitochondrial and peroxisomal disorders. More than 400 IEMs have been identified Lysosomal storage disorders, certain glyco- since 1908.88 The incidence of IEMs is estimated gen storage disorders, and late-onset forms of to be as high as 1 in 1,000 live births.89 Inborn mitochondrial and peroxisomal disorders tend errors of metabolism are caused by single gene to have a chronic insidious progression that defects that result in abnormal synthesis, catabo- results in organ dysfunction and ultimately fail- lism, and/or function of proteins, carbohydrates, ure. Inheritance is autosomal recessive for most fats, organelles (ie, lysosomes, mitochondria, and IEMs (because many are enzyme deficiencies),

Metabolic Disease of the Newborn and Young Child: Inborn Errors of Metabolism 16-21

Copyright © 2012 by the American Academy of Pediatrics and the American College of Emergency Physicians but there are IEMs with autosomal dominant, in gastroenteritis). Unusual dietary preferences, X-linked, and mitochondrial inheritance. particularly protein or carbohydrate aversion, Nearly all states in the United States test and symptoms occurring during diet changes newborns for at least 29, and some for more are often seen in patients with an IEM. By late than 50, genetic diseases, most of which are infancy or early childhood, it is often apparent IEMs.91,92 A table of newborn screening tests that with routine illnesses, children with an IEM performed by state is available online.93 Results become more severely symptomatic and/or re- are usually available within days but might take quire longer to recover than expected. In older weeks. In at least some states, parents can opt children and adults, vigorous exercise, sleep to not have their child screened. False-negative deprivation, hormonal changes, intercurrent newborn screens can result and are most com- illness, anesthesia or surgery, and pregnancy or monly due to screening too soon after birth, delivery commonly precipitate symptoms. weight less than 1,500 g, neonatal illness, medi- A family history of siblings and/or a history cations, transfusions, and problems related to of relatives with similar findings is an important sample collection and handling. Cutoff values clue to possible IEM. A family history of sud- are set to minimize false-negative rates, which den infant death or unexplained neonatal death, results in relatively high false-positive rates. Ne- particularly due to sepsis or neurologic, cardiac, onates who test positive on newborn screens for and/or hepatic dysfunction, should also raise an IEM that can result in acute, life-threatening concern for a possible IEM. Parental consan- decompensation require emergent evaluation.94 guinity increases the likelihood of an autoso- mal recessive IEM. Maternal history of HELLP Clinical Features (hemolysis, elevated liver enzyme, low plate- History let count) during pregnancy is associated with History can vary from abrupt onset of rapidly maternal heterozygosity for fatty acid oxidation progressing illness that results in death within defect and suggests the possibility of homozy- hours, to intermittent recurrent acute episodes gosity for the disease in the child. Decreased of potentially life-threatening decompensation fetal movement has been described with certain in an otherwise well child, to chronic, slow- glycogen and lysosomal storage disorders and ly progressive deterioration throughout de- peroxisomal disorders. A negative family history cades.85,90,95,96 Inborn errors of metabolism that does not rule out an IEM. result in acutely life-threatening critical illness Clinical Manifestations present most commonly within the first year Clinical manifestations of IEMs reflect the mul- of life and usually within the newborn period, tiorgan system involvement typical of these dis- those that cause intermittent decompensation orders. Common clinical presentations include most often manifest later in infancy or early in a sepsis-like illness or cardiac, hepatic, and/or childhood, and those with insidious, progressive neurologic dysfunction.85–87,97–104 Physical ex- symptoms tend to present in older childhood, amination findings are normal for most IEMs. adolescence, or even throughout adulthood. Abnormal odor or hair, hearing deficit, visual In the neonate, the most important clue to an dysfunction, organomegaly, and/or chronic der- IEM is a history of deterioration after an initial matitis should prompt consideration of an IEM. period of apparent good health ranging from Dysmorphic features are seen commonly with hours to weeks.97,98 In neonates and infants, a mitochondrial disorders, peroxisomal disorders, history of poor feeding, vomiting, chronic diar- lysosomal storage, and other disorders of com- rhea, failure to thrive, lethargy, and seizures is plex molecules. The risk of mortality can be very common.99–101 Developmental delay in infants, high, particularly with initial presentation of an particularly with loss of milestones, is strongly undiagnosed IEM. Features of specific IEMs can suggestive of an IEM. Symptoms can be precipi- be found in the texts referenced in this chap- tated by increased intake of protein or carbo- ter85,95,96 and on Web sites, including Online hydrate and/or by a reduction in feeds (eg, as Mendelian Inheritance in Man.88,90

16-22 Metabolic Diseases

Copyright © 2012 by the American Academy of Pediatrics and the American College of Emergency Physicians Neonates retinopathy, optic atrophy, chronic dermato- Inborn errors of metabolism should be consid- ses, skin photosensitivity, nodules, dilated or ered in neonates who are critically ill, particu- hypertrophic cardiomyopathy, cardiac arrhyth- larly those with encephalopathy and/or hepatic mia, jaundice, hepatomegaly, liver dysfunction, dysfunction. Clinical manifestations of IEMs in splenomegaly, pancreatitis, skeletal abnormali- neonates are nonspecific and most commonly ties, hypotonia, dystonia, weakness, periph- include decreased feeding, hiccups, vomiting, eral neuropathy, and/or developmental delay, diarrhea, dehydration, temperature instability, in some cases with loss of milestones. Inborn tachypnea or apnea, , poor perfu- errors of metabolism most likely to present in sion, unusual odor, jaundice, irritability, in- this age group include partial enzyme deficiency voluntary movements or posturing, abnormal urea cycle defects, disorders of carbohydrate tone, seizures, stroke, and/or altered level of metabolism, fatty acid oxidation defects, and consciousness. These same findings are also lysosomal storage disorders. Reye syndrome– manifestations of many other causes, making like symptoms are now recognized often to be the differential diagnosis extensive. Presence of due to IEMs. The differential diagnosis of IEMs one diagnosis does not rule out the possibility in infants and young children with IEMs is ex- of a concomitant IEM. In fact, some IEMs, such tensive and, in addition to that for the neonates, as galactosemia, certain organic aminoacidopa- includes routine illness, cyclic vomiting, diabe- thies, and glycogen storage diseases, are asso- tes, drug or toxin ingestion, malignant tumor, ciated with an increased risk of sepsis. Inborn and Münchausen by proxy. errors of metabolism should be considered in Older Children, Adolescents, and Adults neonates with fetal hydrops or pulmonary or In older children, adolescents, and adults, pre- intracranial hemorrhage in utero or after birth. It viously undiagnosed IEMs usually manifest as should also be recognized that an undiagnosed neurologic and/or psychiatric abnormalities that IEM is a possible cause of sudden infant death, can range from recent to longstanding and from most commonly fatty acid oxidation defects. subtle to severe.103,105 Although rare, the initial The time to onset of symptoms for inborn er- presentation of an IEM can be that of critical rors of substrate and intermediary metabolism illness with rapid progression to death. Manifes- is dependent on significant accumulation of tations of life-threatening IEMs most commonly toxic metabolites, which for severe forms can include profound lethargy, coma, encephalop- occur within hours after the initiation of feed- athy, seizures, stroke, other thromboembolic ing. Neonates with IEMs that result in defects event, cardiac or hepatic dysfunction or fail- in energy production and use are often symp- ure, weakness, and/or paresis. More commonly, tomatic at birth or within the first 24 hours of manifestations are not acute in onset or immi- life. These neonates are less likely to have coma nently life-threatening and include learning as an early manifestation and are more likely disabilities, autism, developmental delay, visual to have dysmorphic features, cardiopulmonary deficits, ocular abnormalities, exercise intoler- compromise, organomegaly, skeletal malforma- ance, muscle cramps, ataxia, abnormal move- tions, and hypotonia. ments, abnormal or aggressive behaviors, and Infants and Young Children psychiatric disturbances, such as anxiety, panic Infants and children with IEMs can have re- attacks, hallucinations, delirium, paranoia, and/ current episodes of vomiting, dehydration, or psychosis. Findings particularly concerning hepatoencephalopathy, ataxia, seizures, stroke, for IEMs include ophthalmologic abnormali- lethargy, and coma but can appear completely ties, progressive neurologic disease, abnormal normal between episodes. Others have dysmor- movements, cerebellar symptoms, and mixed phic features, failure to thrive, microcephaly or psychiatric and neurologic symptoms. Among macrocephaly, alopecia or sparse hair, hearing the most common IEMs on the growing list of and vision deficits, corneal opacities, cataracts, recognized IEMs with late onset are partial orni- subluxed or dislocated lens, cherry red spots, thine transcarbamylase deficiency (particularly

Metabolic Disease of the Newborn and Young Child: Inborn Errors of Metabolism 16-23

Copyright © 2012 by the American Academy of Pediatrics and the American College of Emergency Physicians in females), homocystinuria, certain glycogen storage disorders, mitochondrial and peroxi- YOUR FIRST CLUE somal disorders, and Wilson disease.

Diagnostic Studies Signs and Symptoms of IEMs • History In the acutely ill patient with an undiagnosed - Acute and/or episodic decompensation, condition, results of routine laboratory tests symptoms out of proportion to often provide clues to IEMs and should guide expected with illness 85,86,97,100,106 further evaluation. Most patients - Siblings, other relatives with signs, with acute, life-threatening presentations of symptoms of IEMs, sudden infant death metabolic disease have metabolic acidosis, hy- - Parental consanguinity perammonemia, and/or hypoglycemia. Labora- • Signs, symptoms tory abnormalities can be transient; therefore, - Gastrointestinal: feeding difficulties, specimens should be collected as soon as pos- failure to thrive, vomiting, diarrhea sible, preferably before initiation of any therapy, - Neurologic: irritability, lethargy, including fluids and glucose. If samples were seizures, stroke, ataxia, developmental not collected before treatment, remaining pre- delay, loss of milestones, autism, treatment specimens are likely to be more infor- behavioral abnormalities mative than posttreatment samples and should - Musculoskeletal: abnormal tone, be used if available. A metabolic specialist can muscle weakness, pains, cramps, exercise intolerance be helpful in directing the evaluation of patients with suspected or known IEMs. - Other: dysmorphic features, abnormal hair, hearing, vision deficits, chronic Metabolic Acidosis dermatitis, unusual odor (urine, sweat, Clinical manifestations of metabolic acidosis cerumen breath, saliva) include tachypnea and vomiting. Initial labo- • Common presentations ratory studies to evaluate acid-base status in- - Acute neonatal catastrophe clude electrolytes and blood gas. Laboratory - Neurologic abnormality, dysfunction manifestations of primary metabolic acidosis - Cardiac dysfunction, failure, arrest

are low pH, Pco2, and bicarbonate level and an - Gastrointestinal or hepatic dysfunction, elevated anion gap. Severe metabolic acidosis failure and ketonuria, with or without hyperammone- - Skeletal myopathy mia or hypoglycemia, are hallmarks of organic - Psychiatric disturbance acidemias. An elevated anion gap accompanies acidosis in organic acidemias, mitochondrial disorders, aminoacidopathies, and disorders of carbohydrate metabolism. Lactate and pyruvate should also be obtained in patients with sus- IEMs. For most patients with primary metabolic pected or confirmed acidosis. , acidosis, it is appropriate to test plasma amino although seen in many IEMs, is a nonspecific acids, acylcarnitine profile, urine organic acids, finding in critically ill patients that results from and acylglycines. hypoxia and poor tissue perfusion. An elevated Hyperammonemia lactate:pyruvate ratio (usually >20), provides An ammonia concentration greater than 100 an important clue to determining whether a mcg/dL (60 mcmol/L) in neonates and greater patient has an IEM, most commonly due to a than 80 mcg/dL (50 mcmol/L) in children is mitochondrial or cytoplasmic enzyme disorder. abnormal and potentially neurotoxic. Early Urine with low specific gravity and pH of 5.0 clinical manifestations of hyperammonemia are or higher in the setting of metabolic acidosis tachypnea, anorexia, and irritability. Older indi- suggests renal tubular acidosis, which, although viduals might report headache, abdominal pain, not specific for an IEM, is a feature of many and fatigue. Progression to vomiting, lethargy,

16-24 Metabolic Diseases

Copyright © 2012 by the American Academy of Pediatrics and the American College of Emergency Physicians seizures, coma, and death can occur within acidopathies or mitochondrial disorders might hours. Patients with consistently elevated am- have ketonuria with normal glucose levels. In monia levels can have modestly increased lev- addition to plasma for amino acids and acylcar- els without obvious clinical symptoms. Hepa- nitine profile and urine for organic acids and tomegaly and mild elevation of transaminases acylglycines, serum ketones, plasma free-fatty and/or abnormal clotting factors are other mani- acids, and endocrine studies, including insulin festations. Ammonia concentration is high- and serum cortisol, should be performed. est with urea cycle defects, usually 340–1700 Other Studies mcg/dL (200–1000 mcmol/L) or higher. With Liver function tests, including bilirubin, trans- organic acidemias, ammonia levels do not usu- aminases, and coagulation studies, most com- ally exceed 850 mcg/mL (500 mcmol/L). Lower monly reveal elevated levels in patients with concentrations of ammonia and higher concen- disorders of protein metabolism or fatty acid trations of acids in these patients result in primary oxidation defects but can reveal elevated levels metabolic acidosis, usually with marked ketosis. in IEMs within most categories. Triglycerides With fatty acid oxidation defects, the level of am- and uric acid levels can be elevated in glyco- monia does not usually exceed 425 mcg/dL (250 gen storage disorders. Lactate dehydrogenase, mcmol/L), and acid-base status is usually normal. creatine kinase, and aldolase levels are most Ammonia concentration in the normal range does commonly elevated in patients with disorders not rule out an IEM. To prevent a falsely elevated of carbohydrate metabolism, fatty acid oxida- value, blood for ammonia should be collected tion defects, and mitochondrial disorders and by free flow without a tourniquet and the speci- should be tested in patients with symptoms of men put on ice immediately. In patients with skeletal muscle myopathy and/or with myo- hyperammonemia, plasma for amino acids and globinuria. Urine test results positive for non– acylcarnitine profile and urine for organic acids, glucose-reducing substances suggest possible acylglycine, and orotic acid should be tested. aminoacidopathy or carbohydrate intolerance Hypoglycemia disorder (eg, galactosemia). A serum glucose level less than 40 mg/dL should In select cases, CSF, collected at approxi- be considered abnormally low in neonates and mately the same time as plasma, should be a level less than 40 to 50 mg/dL should be con- frozen for possible measurement of lactate, sidered abnormally low beyond the neonatal pe- pyruvate, and IEM-specific neurometabolites, riod. Clinical findings of hypoglycemia include particularly neurotransmitters. repeated yawning, irritability, pallor, jitteriness, Histologic evaluation of affected tissues, altered mental status, and seizures. Neonates most commonly liver and/or skeletal muscle, can also have a high-pitched cry, hypothermia, and enzyme assay or DNA analysis in leuko- hypotonia, apnea, cyanosis, and poor feeding. cytes, erythrocytes, skin fibroblasts, and liver Older children and adolescents often report or other tissues might be appropriate but are headache, , diaphoresis, pal- not usually emergent. Other studies, includ- pitations, and nausea. Normally, hypoglycemia ing evaluation for bacterial or viral infection, results in ketonuria, except in neonates due radiography, computed tomography, MRI, to a high capacity to use ketones. In neonates, ultrasonography, electrocardiography, and/or significant ketonuria can be an acute manifes- echocardiography, should be obtained emergently tation of an IEM. Beyond the neonatal period, as clinically indicated for evaluation and manage- inappropriately low or absent ketone levels in ment of life-threatening manifestations of IEMs. a patient with hypoglycemia suggest a hyper- In the child who has died, identifying an insulinemic state, a glycogen storage disorder, IEM might still be important because currently carbohydrate intolerance disorder, or fatty acid asymptomatic family members and/or future oxidation defect. The presence of ketones (ke- children might be affected. Samples of plasma, totic hypoglycemia), however, does not rule out serum, urine, and possibly CSF and skin, bile, these disorders. Patients with organic amino- vitreous humor, bone marrow, and selected organ

Metabolic Disease of the Newborn and Young Child: Inborn Errors of Metabolism 16-25

Copyright © 2012 by the American Academy of Pediatrics and the American College of Emergency Physicians THE CLASSICS WHAT ELSE?

Findings of Routine Laboratory Studies Differential Diagnosis of IEM in the in Children With IEMs Critically Ill Patient • Metabolic acidosis • Infection: sepsis, meningitis, encephalitis • Hypoglycemia (hypoketosis or absent • Shock ketones) • Respiratory illness • Hyperammonemia, respiratory • Cardiac disease • Elevated lactate level, lactate:pyruvate • Gastrointestinal illness or obstruction ratio • Hepatic disease • Elevated liver function studies (bilirubin, • Renal disease aspartate aminotransferase, alanine aminotransferase, lactate dehydrogenase) • Neurologic abnormality, disease • Urine-reducing substances, myoglobin • Trauma • Toxin or drug withdrawal • Behavioral, psychiatric disorder

specimens, including skin, heart, liver, kidney, spleen, skeletal muscle, and/or brain, should be Management collected and frozen, preferably, but not limited In addition to rapid assessment and aggressive 86 to, within 1 to 2 hours of death. Consider ob- management of airway, breathing, and circu- taining photographs of patients with dysmorphic lation, goals of treatment of IEMs are to stop features. If the family declines autopsy, consider intake of harmful substances, correct metabol- obtaining permission to collect vitreous humor, ic abnormalities, and eliminate toxic metabo- skin, and/or organ tissue by needle biopsy. lites.85,86,107,108 Even the apparently stable patient with mild symptoms can deteriorate rapidly Differential Diagnosis with progression to death within hours. For an Because the signs and symptoms of inborn er- undiagnosed suspected IEM, this means initiat- rors of metabolism are nonspecific, IEMs are ing treatment empirically as soon as the diag- frequently not considered in the differential di- nosis is considered. For patients with a known agnosis. Therefore, a high index of suspicion is IEM, treatment should be disease and patient important because the physician must initiate specific. Families should have an emergency appropriate therapy without delay (and without treatment plan specifically for their child devel- a final diagnosis in hand) to decrease morbidity oped by their IEM specialist with them or avail- and mortality. A concomitant acquired disorder able electronically. In addition, families might can obscure the diagnosis of an inherited meta- have a plan detailing their desired interventions bolic disease condition. For example, neutrope- should lifesaving resuscitation be necessary. nia can occur in organic acidemias, leading to Once airway, breathing, ventilation, and an increased susceptibility of bacterial infection. vascular access have been established, hydra- The underlying disorder might be missed if an tion, glucose as necessary to correct hypogly- infection is also present and focus is directed cemia and prevent catabolism, and therapy to solely toward antimicrobial therapy. Escherichia correct acidosis and hyperammonemia must coli sepsis frequently supervenes in neonates be initiated. Protocols for treatment of acute with galactosemia, and the typical jaundice, illness in patients with a suspected IEM and liver failure, and vomiting that accompany that some diagnosed IEMs are available on the New disorder might wrongly be ascribed solely to England Consortium Web site.107 Consultation sepsis as well. with a metabolic specialist should be initiated

16-26 Metabolic Diseases

Copyright © 2012 by the American Academy of Pediatrics and the American College of Emergency Physicians if available. With appropriate therapy, patients For patients with an aminoacidopathy, organic can recover completely without sequelae. acidemia, or fatty acid oxidation defect, hyper- Hydration is critical not only to restore in- ammonemia usually improves with rehydration travascular volume but also to promote urinary and the reversal of acidosis and hypoglycemia. excretion of toxic metabolites. For IEMs that For severe hyperammonemia in patients with are potentially acutely life-threatening, fluid bo- urea cycle defects, hemodialysis is the mainstay luses should be administered as 10% dextrose of treatment. Extracorporeal membrane oxygen- normal saline unless the patient is hypoglyce- ation–assisted hemodialysis is the most effective mic, in which case a glucose bolus is indicated form of hemodialysis but has increased risks, (0.5 g/kg as 10% dextrose for neonates and 10% particularly in neonates. Exchange transfusion or 25% dextrose for children; note the “50 rule” is not effective. Recommendations of the New described earlier). Preferentially, colloids should England Consortium are to consider dialysis for be avoided because they increase nitrogen load. ammonia levels greater than 300 mcg/dL (175 After bolus fluid, maintenance fluid should be mcmol/L) and transfer to a facility or unit with administered as 10% to 12.5% dextrose half dialysis capability for ammonia levels greater normal saline at a rate high enough to prevent than 200 to 250 mcg/dL (120–150 mcmol/L). catabolism by maintaining serum glucose at If dialysis is not immediately available for am- 120 to 170 mg/dL. Large fluctuations in serum monia levels between 100 and 200 mcg/dL glucose should be avoided. Although contro- (60–175 mcmol/L), sodium phenylacetate versial, 0.1 to 0.2 U/kg per hour of insulin can and sodium benzoate (available as Ammonul) be administered to prevent hyperglycemia. For should be administered to patients with known a suspected IEM, stop all oral intake to ensure or suspected urea cycle defect to augment ni- that potentially harmful protein or sugars are trogen excretion.109 The combination of sodium avoided. For a known IEM, dietary restriction phenylacetate and sodium benzoate is approved should be disease specific. as adjunctive therapy for acute hyperammone- Treatment of acidosis with bicarbonate is mia and associated encephalopathy due to urea thought to be indicated for IEMs because of on- cycle defects but is also used to treat hyperam- going acid production; however, data regarding monemia of unknown origins. Ondansetron the degree of acidosis for which it should be should be administered with sodium phenyl- initiated and dose are lacking. Recommenda- acetate and sodium benzoate. Potassium, which tions for administration of bicarbonate range can be depleted by sodium phenylacetate and from pH 7.0 to 7.2 and serum bicarbonate as sodium benzoate, should be monitored and re- low as 14 to 16 mEq/L. The recommended dose placed as potassium acetate, 0.5 to 1 mEq/kg range is 0.25 to 2 mEq/kg per hour, depending per hour, as needed. In addition, 10% arginine in part on the specific IEM. Bicarbonate should hydrochloride, which enhances urea cycle activ- be administered cautiously in patients with hy- ity, should be given for treatment of urea cycle perammonemia because alkalinization converts defects with the exception of arginase deficiency.

NH4+ to NH3, which is less readily excreted in Citrulline can be administered to augment ni- urine and more readily crosses the blood-brain trogen clearance in urea cycle defects with the barrier, increasing the risk of brain complica- exception of argininosuccinic acid synthetase tions. If administering sodium bicarbonate, po- (citrullinemia) and argininosuccinase acid lyase tassium should be given as potassium acetate. deficiencies. Although data are limited, recent For severe, intractable metabolic acidosis, he- literature suggests hypertonic saline should be modialysis should be considered. Acidosis must given for cerebral edema rather than mannitol. be corrected cautiously to minimize paradoxical Corticosteroids increase catabolism and there- effects on the CNS that can be caused by rapid fore should not be given. To assess patient re- or overcorrection. sponse to treatment, glucose, acid-base status, Hyperammonemia can be acutely life- ammonia, and electrolytes should be monitored threatening and must be treated immediately. serially.

Metabolic Disease of the Newborn and Young Child: Inborn Errors of Metabolism 16-27

Copyright © 2012 by the American Academy of Pediatrics and the American College of Emergency Physicians Appropriate antibiotics should be given if oxidation disorders but is usually used after there is concern about possible serious bacte- consultation with a metabolic specialist. The rial infection. As necessary, fresh frozen plasma child might need immediate admission to an should be administered to treat coagulopathy. intensive care unit for further monitoring and Intravenous l-carnitine, 25 to 50 mg/kg, can other procedures (eg, hemodialysis). be administered empirically in life-threatening In children who are already known to have situations associated with known or suspected an IEM, the importance of listening closely to carnitine deficiency (eg, primary carnitine defi- the history provided by the parents or regular ciency, certain organic aminoacidopathies, and caregiver must be stressed. A child with a meta- fatty acid oxidation defects); however, because bolic disorder can appear relatively well but it can be harmful with some IEMs, consultation might decompensate rapidly. If such a child is not with an IEM specialist is highly recommended. maintaining adequate fluid and caloric intake, Carnitine should not be administered with so- “does not seem to be his usual self,” or both, the dium phenylacetate and sodium benzoate. In- metabolic specialist should be contacted, and IV

travenous pyridoxine (B6), 100 mg; IV folate, fluids with dextrose should be administered even 2.5 mg; and/or biotin, 10 to 40 mg, orally or via in the absence of documented hypoglycemia or nasogastric tube can be given empirically to neo- other markers of metabolic imbalance. The goal nates with seizures unresponsive to conventional in such cases is to provide therapy in a timely anticonvulsants to treat a possible vitamin or co- manner to prevent a metabolic crisis. factor responsive IEM. Although there are other disease-specific adjunctive therapies, administra- tion of these agents in the ED is rarely indicated. THE BOTTOM LINE Rapid assessment, establishment of vascu- lar access, and administration of appropriate Issues in the Diagnosis and fluids (with glucose) are priorities in treating Management of IEMs patients with IEMs in acute crisis. Dehydration • Maintain a high index of suspicion, and hypoglycemia should be treated with bo- especially in the presence of luses of normal saline and dextrose solutions, hypoglycemia, metabolic acidosis, respectively. Maintenance IV fluids should then hyperammonemia, and or inappropriate be started to prevent recurrence of hypoglyce- ketosis. mia and hypovolemia and stop the catabolic • Although specialized tests are needed to arrive at a final diagnosis, simple spiral. Administration of 10% dextrose (with laboratory studies often provide the first electrolytes based on the child’s weight) at 1.5 clues to the presence of an IEM. Obtain times maintenance is usually adequate. General pretreatment specimens when possible. supportive measures (maintenance of oxygen- • Early detection and treatment are ation and acid/base balance and treatment of essential to optimize immediate and infection) should, of course, be provided. In- long-term outcome. travenous carnitine (100 mg/kg) can be benefi- cial in certain organic acidemias and fatty acid

16-28 Metabolic Diseases

Copyright © 2012 by the American Academy of Pediatrics and the American College of Emergency Physicians KEY POINTS

Initial Treatment of Patients With IEMs • Evaluation - Contact metabolic specialist immediately once diagnosis is suspected. - Obtain specimens, ideally before treatment.

* Blood: venous blood gas, glucose, electrolytes, BUN, creatinine, complete blood cell count * If lethargic, ammonia (free flow, ice) * Lactate dehydrogenase, aldolase, creatinine kinase if skeletal muscle myopathy is suspected * Lactate, pyruvate if acidosis (free flow, special perchloric acid tube for pyruvate) * Plasma amino acids, acylcarnitine profile if initial evaluation supports possible IEM * Urine: urinalysis, urine culture if concern of infection; urine amino, organic acids, acylglycine if initial evaluation supports possible IEM • Treatment - Stop all oral feedings - Fluid hydration with 10% dextrose normal saline - Glucose to treat hypoglycemia, prevent catabolism (use the “50 rule”) - IV bicarbonate corrects acidosis but could worsen hyperammonemia equilibrium - For hyperammonemia, plan for detoxification by hemodialysis and/or sodium phenylacetate and sodium benzoate (Ammonul) and arginine (except for arginase deficiency); if using sodium phenylacetate and sodium benzoate, administer ondansetron and treat hypokalemia with potassium acetate

- Seizures: IV pyridoxine (B6), 100 mg; IV folate, 2.5 mg; and/or biotin, 10 to 40 mg, orally or via nasogastric tube - Suspected carnitine deficiency: IV l-carnitine, 25 to 50 mg/kg

Metabolic Disease of the Newborn and Young Child: Inborn Errors of Metabolism 16-29

Copyright © 2012 by the American Academy of Pediatrics and the American College of Emergency Physicians CHAPTER REVIEW

Check Your Knowledge 5. Of the following inborn errors of metabolism (IEMs), which is most likely 1. Which of the following is most associated to present with an insidious onset and with the development of cerebral edema progressive worsening? in a child with diabetic ketoacidosis A. Organic acidemias (DKA)? B. Aminoacidopathies A. Ketosis C. Fatty acid oxidation defects B. High fluid administration (20 mL/kg D. Lysosomal storage disorders per hour) 6. Laboratory abnormalities due to C. Urinary tract infection IEMs commonly include which of the D. Treatment with sodium bicarbonate following (best choice)? 2. Which of the following is a prominent A. Metabolic acidosis, hyperammonemia, finding in female newborns with hypoglycemia congenital adrenal hyperplasia? B. Hypoglycemia, ketosis, leukopenia A. Acanthosis nigricans C. Hematuria, azotemia, B. Ambiguous genitalia hypercholesterolemia C. Hyperpigmentation D. Proteinuria, hypercholesterolemia, D. Midline facial defects and hypoalbuminemia microphallus E. Hypocalcemia, hyponatremia, 3. Syndrome of inappropriate secretion of hypokalemia antidiuretic hormone should be treated 7. The most appropriate initial rapid fluid with: infusion for a patient with a suspected A. intravenous hydrocortisone. IEM who is tachycardic, hypoglycemic, B. oral sodium supplements. and acidotic is: C. hypertonic saline, 10-mL/kg infusion A. normal saline (20 mL/kg). if the sodium concentration is 130 B. lactated Ringer solution (20 mL/kg). mEq/L. C. 10% Dextrose normal saline (20 mL/ D. fluid restriction unless the patient kg). is seizing or severely lethargic or D. 25% Dextrose (2 mL/kg) and normal comatose. saline (20 mL/kg). E. sodium restriction. E. 10% Dextrose (2 mL/kg) and normal 4. Which of the following statements saline (20 mL/kg). regarding hypernatremia in patients with 8. The most effective treatment for severe diabetes insipidus (DI) is correct? hyperammonemia due to an IEM is: A. Never occurs in infants with DI A. hemodialysis. because of high fluid intake B. exchange transfusion. B. Occurs in all patients with DI C. peritoneal dialysis. C. Rare in older children with DI and D. sodium phenylacetate and sodium normal thirst mechanisms benzoate (Ammonul). D. Should be treated with 3% sodium E. hyperventilation. chloride solution

16-30 Metabolic Diseases

Copyright © 2012 by the American Academy of Pediatrics and the American College of Emergency Physicians CHAPTER REVIEW

21. Swinnen SG, Mullins P, Miller M, Hoekstra JB, Holleman F. References Changing the glucose cut-off values that define hypoglycaemia 1. Levine B, Anderson B, Butler D, Antisdel J, Brackett J, Laffel L. has a major effect on reported frequencies of hypoglycaemia. Predictors of glycemic control and short-term adverse outcomes Diabetologia. 2009;52:38-41. in youth with type 1 Diabetes. J Pediatr. 2001;39:197-203. 22. Amiel SA, Dixon T, Mann R, Jameson K. Hypoglycaemia in type 2. Rewers A, Chase H, Mackenzie T, et al. Predictors of acute 2 diabetes. Diabet Med. 2008;25:245-254. complications in children with type 1 Diabetes. JAMA. 23. Cryer, PE. Preventing hypoglycaemia: what is the appropriate 2002;287:2511-2518. glucose alert value? Diabetologia. 2009;52:35-37. 3. Rewers A, Klingensmith G, Davis C, et al. Presence of diabetic 24. Rewers A, Chase HP, Mackenzie T, et al. Predictors of acute ketoacidosis at diagnosis of diabetes mellitus in youth: the Search complications in children with type 1 diabetes. JAMA. for Diabetes in Youth Study. Pediatrics. 2008;121:e1258-e1266. 2002;287:2511-2518. 4. Flood R, Chiang V. Rate and prediction of infection in children 25. Davis EA, Keating B, Byrne GC, Russell M, Jones TW. Impact of with diabetic ketoacidosis. Am J Emerg Med. 2001;19:270-273. improved glycaemic control on rates of hypoglycaemia in insulin 5. Glaser N, Barnett P, McCaslin I, et al. Risk factors for cerebral dependent diabetes mellitus. Arch Dis Child. 1998;78:111-115. edema in children with diabetic ketoacidosis. N Engl J Med. 26. Nimri R, Weintrob N, Benzaquen H, Ofan R, Fayman G, Phillip 2001;344:264-269. M. Insulin pump therapy in youth with type 1 diabetes: a 6. Sperling M, Dunger D, Acerini C, et al. ESPE/LWPES consensus retrospective paired study. Pediatrics. 2006;117:2126-2131. statement on diabetic ketoacidosis in children and adolescents. 27. Berhe T, Postellon D, Wilson B, Stone R. Feasibility and safety of Pediatrics. 2003;113:e133-e140. insulin pump therapy in children aged 2 to 7 years with type 1 7. Wolfsdorf J, Glaser N, Sperling M. Diabetic ketoacidosis in diabetes: a retrospective study. Pediatrics. 2006;117:2132-2137. infants, children and adolescents: a consensus statement from the 28. Colino E, Lopez-Capape M, Golmayo L, Alvarez MA, Alonso American Diabetes Association. Diab Care. 2006;29:1150-1159. M, Barrio R. Therapy with insulin glargine (Lantus) in toddlers, 8. Gutierrez J, Bagatell R, Sampson M, Theodorou A, Berg R. children and adolescents with type 1 diabetes. Diabetes Res Clin Femoral central venous catheter-associated deep venous Pract. 2005;70:1-7. thrombosis in children with diabetic ketoacidosis. Crit Care Med. 29. Alemzadeh R, Berhe, Wyatt DT. Flexible insulin therapy with 2003;31:80-83. glargine insulin improved glycemic control and reduced severe 9. Worly J, Fortenberry J, Hansen I, Chambliss C, Stockwell J. Deep hypoglycemia among preschool-aged children with type 1 venous thrombosis in children with diabetic ketoacidosis and diabetes mellitus. Pediatrics. 2005;115:1320-1324. femoral central venous catheters. Pediatrics. 2004;113:e57-e60. 30. Rewers A, Chase HP, Mackenzie T, et al. Predictors of acute 10. Zeitler P, Haaq A, Rosenbloom A, Glaser N. Hyperglycemic complications in children with type 1 diabetes. JAMA. hyperosmolar syndrome in children: pathophysiologic 2002;287:2511. considerations and suggested guidelines for treatment. J Pediatr. 31. Cryer PE. The barrier of hypoglycemia in diabetes. Diabetes. 2011;158:9-14. 2008;57:3169-3176. 11. Glaser N, Marcin J, Wooton-Gorges S, et al. Correlation of 32. Mukamel M, Weitz R, Nissenkorn I, Yassur I, Varsano I. Acute clinical and biochemical findings with DKA-related cerebral cortical blindness associated with hypoglycemia. J Pediatr. edema in children using magnetic resonance diffusion weighted 1981;98:583-584. imaging. J Pediatr. 2008;153:541-546. 33. Garty BZ, Dinari G, Nitzan M. Transient acute cortical blindness 12. Koves I, Neutze J, Donath S, et al. The accuracy of clinical associated with hypoglycemia. Pediatr Neurol. 1987;3:169-170. assessment of dehydration during diabetic ketoacidosis in 34. Weinstein DA, Butte AJ, Raymond K, Korson MS, Weiner childhood. Diab Care. 2004;27:2485-2487. DL, Wolfsdorf JI. High incidence of unrecognized metabolic 13. Green S, Rothrock S, Ho J, et al. Failure of adjunctive bicarbonate and endocrinologic disorders in acutely ill children with to improve outcome in severe pediatric diabetic ketoacidosis. hypoglycemia. Pediatr Res. 2001;49:88A. Ann Emerg Med. 1998;31:41-48. 35. Losek JD. Hypoglycemia and the ABCs (sugar) of pediatric 14. Buckingham B, Roe T, Yoon J. Rhabdomyolysis in diabetic resuscitation. Ann Emerg Med. 2000;35:43-46. ketoacidosis. Am J Dis Child. 1981;135:352-354. 36. Lteif AN, Schwenk WF. Hypoglycemia in infants and children. 15. Shilo S, Werner D, Hershko C. Acute hemolytic anemia caused Endocrinol Metabol Clin. 1999;28:620-646. by severe hypophosphatemia in diabetic ketoacidosis. Acta 37. White PC, Speiser PW. Congenital adrenal hyperplasia due to Haemat. 1985;73:55-57. 21-hydroxylase deficiency.Endocr Rev. 2000;21:245-291. 16. Marcin J, Glaser N, Barnett P, et al. Clinical and therapeutic 38. Arlt W, Allollo B. Adrenal insufficiency.Lancet. 2003;361:1881- factors associated with adverse outcomes in children with DKA- 1893. related cerebral edema. J Pediatr. 2003;141:793-797. 39. Root AW, Shulman DI. Clinical adrenal disorders. In: Pescovitz 17. Workgroup on Hypoglycemia, American Diabetes Association. OH, Eugster EA, eds. Pediatric Endocrinology, Mechanisms, Defining and reporting hypoglycemia in diabetes: a report from Manifestations, and Management. Philadelphia, PA: Lippincott the American Diabetes Association Workgroup on Hypoglycemia. Williams & Wilkins; 2004:568-600. Diabetes Care. 2005;28:1245-1249. 40. Wilson TA, Speiser P. Adrenal insufficiency. www.emedicine.com/ 18. Clarke W, Jones T, Rewers A, Dunger D, Klingensmith GJ. PED/topic47.htm. Accessed April 30, 2006. Assessment and management of hypoglycemia in children and 41. Shaikh MG, Lewis P, Kirk JM. Thyroxine unmasks Addison’s adolescents with Diabetes. Pediatr Diabetes. 2009;10(suppl disease. Acta Paediatr. 2004;93:1663-1665. 12):134-145. 42. Havard CW, Saldanha VF, Bird R, Gardner R. Adrenal function in 19. Cryer PE, Axelrod L, Grossman AB, et al. Evaluation and hypothyroidism. BMJ. 1970;1:337-339. management of adult hypoglycemic disorders: an Endocrine 43. Giavoli C, Libe R, Corbetta S, et al. Effect of recombinant human Society Clinical Practice Guideline. J Clin Endocrinol Metab. growth hormone (GH) replacement on the hypothalamic- 2009;94:709-728. pituitary-adrenal axis in adult GH-deficient patients.J Clin 20. Frier BM. Defining hypoglycaemia: what level has clinical Endocrinol Metab. 2004;89:5397-5401. relevance? Diabetologia. 2009;52:31-34.

Chapter Review 16-31

Copyright © 2012 by the American Academy of Pediatrics and the American College of Emergency Physicians CHAPTER REVIEW

44. den Brinker M, Joosten KF, Liem O, et al. Adrenal insufficiency 60. Maghnie M, Cosi G, Genovese E, et al. Central diabetes insipidus in meningococcal sepsis: bioavailable cortisol levels and impact in children and young adults. N Engl J Med. 2000;343:998-1007. of interleukin-6 levels and intubation with etomidate on adrenal 61. Alter CA, Bilaniuk LT. Utility of magnetic resonance imaging function and mortality. J Clin Endocrinol Metab. 2005;90:5110- in the evaluation of the child with central diabetes insipidus. J 5117. Pediatr Endocrinol Metab. 2002;15(suppl 2):681-687. 45. Oberfield SE, Chin D, Uli N, David R, Sklar C. Endocrine late 62. Ghirardello S, Garrè ML, Rossi A, Maghnie M. The diagnosis effects of childhood cancers. J Pediatr. 1997;131(1 pt 2):S37-S41. of children with central diabetes insipidus. J Pediatr Endocrinol 46. Bottner A, Keller E, Dratzsch J, Stobbe H, Weigel JF, Keller A. Metab. 2007;20:359-375. PROP 1 mutations cause progressive deterioration of anterior 63. Brooks BS, el Gammal T, Allison JD, et al. Frequency and pituitary function including adrenal insufficiency: a longitudinal variation of the posterior pituitary bright signal on MR images. analysis. J Clin Endocrinol Metab. 2004;89:5256-5265. AJNR Am J Neuroradiol. 1989;10:943-948. 47. Benvenga S, Campenni A, Ruggeri RM, Trimarchi F. Clinical 64. Alonso G, Bergada I, Heinrich JJ. Magnetic resonance imaging in review 113: hypopituitarism secondary to head trauma. J Clin central diabetes insipidus in children and adolescents: findings at Endocrinol Metab. 2000;85:1353-1361. diagnosis and during follow-up. An Esp Pediatr. 2000;53: 48. Shulman DI, Palmert MR, Kemp SF; Lawson Wilkins Drug 100-105. and Therapeutics Committee. Adrenal insufficiency: still 65. Kirchlechner V, Koller DY, Seidl R, et al. Treatment of a cause of morbidity and death in childhood. Pediatrics. nephrogenic diabetes insipidus with hydrochlorothiazide and 2007;119:e484-e494. amiloride. Arch Dis Child. 1999;80:548-552. 49. Cheetham T, Baylis PH. Diabetes insipidus in children: 66. van Lieburg AF, Knoers NV, Monnens LA. Clinical presentation pathophysiology, diagnosis and management. Paediatr Drugs. and follow-up of 30 patients with congenital nephrogenic 2002;4:785-796. diabetes insipidus. J Am Soc Nephrol. 1999;10:1958-1964. 50. Verbalis JG. Diabetes insipidus. Rev Endocr Metab Disord. 67. Alon U, Chan JC. Hydrochlorothiazide-amiloride in the 2003;4:177-185. treatment of congenital nephrogenic diabetes insipidus. Am J 51. Mahone C, Weinberger E, Bryant C, Ito M, Jameson, JL, Ito M. Nephrol. 1985;5:9-13. Effects of aging on vasopressin production in a kindred with 68. Knoers N, Monnens LA. Amiloride-hydrochlorothiazide autosomal dominant neurohypophyseal diabetes insipidus due versus indomethacin hydrochlorothiazide in the treatment of to the delta-E47 neurophysin mutation. J Clin Endocr Metab. nephrogenic diabetes insipidus. J Pediatr. 1990;117:499-502. 2002;87:870-876. 69. Bartter FC, Schwartz WB. The syndrome of inappropriate 52. Bichet DG, Birnbaumer M, Lonergan M, et al. Nature and secretion of antidiuretic hormone. Am J Med. 1967;42:790-806. recurrence of AVPR2 mutations in X-linked nephrogenic diabetes 70. Verbalis JG, Goldsmith SR, Greenberg A, et al. Hyponatremia insipidus. Am J Hum Genet. 1994;55:278-286. treatment guidelines 2007: expert panel recommendations. Am J 53. Baylis PH, Cheetham T. Diabetes insipidus. Arch Dis Child. Med. 2007;120(11 suppl 1):S1-S21. 1998;79:84-89. 71. Baylis PH. The syndrome of inappropriate antidiuretic hormone 54. Maghnie M, Cosi G, Genovese E, et al. Central diabetes insipidus secretion. Int J Biochem Cell Biol. 2003;35:1495-1499. in children and young adults. N Engl J Med. 2000;343:998-1007. 72. Bartter FC, Schwartz WB. The syndrome of inappropriate 55. Mootha SL, Barkovich AJ, Grumbach MM, et al. Idiopathic secretion of antidiuretic hormone. Am J Med. 1967;42:790-806. hypothalamic diabetes insipidus, pituitary stalk thickening, and 73. Cogan E, Debieve MF, Pepersack T, et al. Natriuresis and atrial the occult intracranial germinoma in children and adolescents. J natriuretic factor secretion during inappropriate antidiuresis. Am Clin Endocrinol Metab. 1997;82:1362-1367. J Med. 1988;84(3 pt 1):409-418. 56. Rutishauser J, Kopp P, Gaskill MB, et al. Clinical and 74. Albanese A, Hindmarsh P, Stanhope R. Management of molecular analysis of three families with autosomal dominant hyponatremia in patients with acute cerebral insults. Arch Dis neurohypophyseal diabetes insipidus associated with a novel and Child. 2001;85:246-251. recurrent mutations in the vasopressin-neurophysin II gene. Eur J 75. Huang EA, Feldman BJ, Schwartz ID, Geller DH, Rosenthal SM, Endocrinol. 2002;146:649-656. Gitelman SE. Oral urea for the treatment of chronic syndrome of 57. Morello JP, Bichet DG. Nephrogenic diabetes insipidus. Annu Rev inappropriate antidiuresis in children. J Pediatr. 2006;148:128-131. Physiol. 2001;63:607-630. 76. Ophir E, Solt I, Odeh M, Bornstein J. Water intoxication-a 58. Robinson AG, Verbalis JG. Posterior pituitary gland. In: Larsen dangerous condition in labor and delivery rooms. Obstet Gynecol PR, Kronenberg HM, Melmed S, et al, eds. Williams Textbook of Surv. 2007;62:731-738. Endocrinology. 10th ed. Philadelphia, PA: Saunders; 2003:281-329. 77. Jose CJ, Perez-Cruet J. Incidence and morbidity of self-induced 59. Muglia LJ, Majzoub JA. Disorders of the posterior pituitary. water intoxication in state mental hospital patients. Am J In: Sperling MA, ed. Pediatric Endocrinology. Philadelphia, PA: Psychiatry. 1979;136:221. Saunders; 2002. 78. Illowsky BP, Kirch DG. Polydipsia and hyponatremia in psychiatric patients. Am J Psychiatry. 1988;145:675.

16-32 Metabolic Diseases

Copyright © 2012 by the American Academy of Pediatrics and the American College of Emergency Physicians CHAPTER REVIEW

79. de Leon J. Polydipsia—a study in a long-term psychiatric unit. 94. ACGME Newborn Screening Work Group: Levy HL, Watson, Eur Arch Psychiatry Clin Neurosci. 2003;253:37. MS, Metabolic Disorders: Berry G, Goodman S, Marsden D, et 80. Goldman MB, Luchins DJ, Robertson GL. Mechanisms of altered al. Newborn Screening Act Sheet and Confirmatory Algorithms. water metabolism in psychotic patients with polydipsia and http://www.acmg.net/resources/policies/ACT/condition-analyte- hyponatremia. N Engl J Med. 1988;318:397. links.htm. Accessed September 25, 2010. 81. Thompson CJ, Edwards CR, Baylis PH. Osmotic and non-osmotic 95. Clarke JTR. A Clinical Guide to Inherited Metabolic Diseases. 3rd ed. regulation of thirst and vasopressin secretion in patients with Cambridge, England: Cambridge University Press; 2006. compulsive water drinking. Clin Endocrinol (Oxf). 1991;35:221-228. 96. Hoffmann GF, Zschocke J, Nyhan WL. Inherited Metabolic 82. Rao KJ, Miller M, Moses A. Water intoxication and thioridazine. Diseases: A Clinical Approach. New York, NY: Springer; 2006. Ann Intern Med. 1975;82:61. 97. Goodman SI. Inherited metabolic disease in the newborn: 83. Farrar HC, Chande VT, Fitzpatrick DF, Shema SJ. Hyponatremia approach to diagnosis and treatment. Adv Pediatr. 1986;33: as the cause of seizures in infants: a retrospective analysis of 197-223. incidence, severity and clinical predictors. Ann Emerg Med. 98. Enns GM, Packman S. Diagnosing inborn errors of metabolism in 1995;26:42-48. the newborn: clinical features. Neo Rev. 2001;2000:e183-e190. 84. Arn PH, Valle DL, Brusilow SW. Inborn errors of metabolism: not 99. Ward JC. Inborn errors of metabolism of acute onset in infancy. rare, not hopeless. Contemp Pediatr. 1988;5:47-63. Pediatr Rev. 1990;11:205-216. 85. Fernandes J, Saudubray JM, van den Berghe G, Walter JH, 100. Burton BK. Inborn errors of metabolism in infancy: a guide to eds. Inborn Metabolic Diseases: Diagnosis and Treatment. 4th ed. diagnosis. Pediatrics. 1998;102:e69. Heidelberg, Germany: Springer Medizin Verlag; 2006. 101. Ficicioglu C, Haack K. Failure to thrive: when to suspect inborn 86. Weiner DL. Metabolic emergencies. In: Fleisher GR, Ludwig S. errors of metabolism. Pediatrics. 2009;124:972-975. Textbook of Pediatric Emergency Medicine. 6th ed. Philadelphia, PA: 102. Calvo M, Artuch R, Macia E, et al. Diagnostic approach to inborn Lippincott Williams & Wilkins; 2010:972-991. errors of metabolism in an emergency unit. Pediatr Emerg Care. 87. Kwon KT, Tsai VW. Metabolic emergencies. Emerg Med Clin North 2000;16:405-408. Am. 2007;25:1041-1060. 103. Calvin J, Hogg S. Biochemical investigation for inborn errors of 88. McKusik VA. OMIM Online Mendelian Inheritance in Man metabolism in adults presenting with neurological disorders. Adv [database online]. McKusick-Nathans Institute for Genetic Clin Neurosci Rehabil. 2010;9:12-16. Medicine, Johns Hopkins University, Baltimore, MD, and 104. Mannenbach MS. Children with Inborn Errors of Metabolism: National Center for Biotechnology Information, National Library Recognizing the Unusual and Life-threatening. AllBusiness.com. of Medicine, Bethesda, MD; 2000. http://www.ncbi.nlm.nih.gov/ LexisNexis: 2010. http://www.allbusiness.com/print/12780095- omim. Accessed September 26, 2010. 1-22eeq.html Accessed September 25, 2010. 89. Ellaway C, Wilcken B, Christodoulou J. Clinical approach to 105. Gray RGF, Preece MA, Green SH, Whitehouse W, Winer J, inborn errors of metabolism presenting in the newborn period. J Green A. Inborn errors of metabolism as a cause of neurological Paediatr Child Health. 2002;38:511-517. disease in adults: an approach to investigation. J Neurol Neurosurg 90. Valle, D, Beaudet, AL, Vogelstein B, Kinzler KW, Antonarakis SE, Psychiatry. 2000;69:5-12. Ballabio A. The Online Metabolic and Molecular Bases of Inherited 106. Blau N, Duran M, Blaskovics ME, Gibson KM. Physician’s Guide to Disease. New York, NY: McGraw-Hill; 2007. http://www. the Laboratory Diagnosis of Metabolic Diseases. 2nd ed. New York, ommbid.com/. Accessed April 11, 2011. NY: Springer; 2002. 91. Chace DH, Kalas TA, Naylor EW. Use of tandem mass 107. Acute Illness Protocols. New England Consortium of Metabolic spectrometry for multianalyte screening of dried blood Programs. http://newenglandconsortium.org/for-professionals/ specimens from newborns. Clin Chem. 2003;49:1797-1817. acute-illness-protocols/. Accessed September 25, 2010. 92. Kaye CI; American Academy of Pediatrics, Committee 108. Ogier de Baulny H. Management and emergency treatments of on Genetics. Newborn screening fact sheets. Pediatrics. neonates with a suspicion of inborn errors of metabolism. Semin 2006;118:e934-e963. www.pediatrics.org/cgi/content/full/118/3/ Neonatol. 2002;7:17-26. e934. Accessed September 25, 2010. 109. Ammonul package insert. http://www.medicis.com/products/pi/ 93. Newborn Screening Status Report. National Newborn Screening pi_ammonul.pdf. Accessed April 11, 2011. and Genetics Resource Center (NNSGRC). United States table of newborn screening tests performed by state. http://genes-r-us. uthscsa.edu/nbsdisorders.pdf. Accessed September 25, 2010. International screening programs. http://genes-r-us.uthscsa.edu/ resources/newborn/international.htm. Accessed September 25, 2010.

Chapter Review 16-33

Copyright © 2012 by the American Academy of Pediatrics and the American College of Emergency Physicians CHAPTER REVIEW

1 A 10-year-old boy presents to the emergency department (ED) with altered mental status and a 2-week history of polyuria and weight loss. He is experiencing abdominal pain. On examination, he has a respiratory rate of 36/min, a heart rate of 150/min, a systolic blood pressure of 80 mm Hg, and a temperature of 37.9°C (100.2°F). Pulse oximetry oxygen saturation is 97% on room air. On examination, he appears lethargic, has dry mucous membranes, has normal abdominal examination results, and has CASE SUMMARY SUMMARY CASE nonfocal neurologic examination results. Bedside rapid glucose measurement is above the upper limit of the glucose meter (>500 mg/dL).

1. What therapeutic actions must be taken immediately? 2. What laboratory tests would you order? 3. What is the most important potential complication?

This child is hypotensive, mainly due to dehydration. Immediate volume resuscitation with boluses of normal saline (20 mL/kg) should be administered to restore perfusion. An insulin infusion should be initiated as soon as possible after intravenous fluid therapy has been initiated.

A reasonable set of initial laboratory tests would include measurement of serum electrolytes, a venous blood gas analysis, and urinalysis for ketones. Complete blood cell counts are not usually helpful.4 This child must be closely monitored for the development or progression of cerebral edema. Clinically apparent cerebral edema occurs in approximately 1% of children with DKA. Therefore, frequent neurologic monitoring is indicated for all children with DKA.

2 A 14-month-old girl presents to the ED with a 1-day history of tactile fever, loss of appetite, and intermittent emesis. This morning, she did not seem interested in her usual breakfast and seemed sleepier than usual. On examination, she has respirations of 40/min, a heart rate of 170/min, blood pressure of 80/40 mm Hg, and a temperature of 36.0°C (96.8°F). She appears lethargic and has mildly dry mucous membranes and no obvious source of infection. Bedside rapid glucose measurement reveals a blood CASE SUMMARY SUMMARY CASE glucose level of 20 mg/dL.

1. What therapeutic measures must be taken immediately? 2. What laboratory tests would you order? 3. What complications can occur as part of the disease process and secondary to therapeutic intervention?

16-34 Metabolic Diseases

Copyright © 2012 by the American Academy of Pediatrics and the American College of Emergency Physicians CHAPTER REVIEW

The child is hypoglycemic and likely volume depleted. Resuscitation with boluses

CONT. of 2 mL/kg of 25% dextrose (or 5 mL/kg of 10% dextrose) to normalize blood glucose

levels should be started without delay, followed by administration of volume (normal 2 saline bolus of 20 mL/kg) until perfusion is restored. Fluids consisting of 10% dextrose and 0.45% sodium chloride at 1.5 times maintenance should be started. Appropriate potassium supplementation is added as needed. Initial studies include measurement of electrolytes, measurement of glucose, blood gas analysis, complete blood cell count, blood and urine cultures, and urinalysis for ketones. If possible, samples should be

CASE SUMMARY SUMMARY CASE obtained before initiation of therapy, including administration of glucose and fluids. The finding of nonketotic (hypoketotic) hypoglycemia is a clue to the presence of an underlying fatty acid oxidation defect. Ammonia levels might be mildly elevated. Metabolic disorders might be unmasked by an infection severe enough to cause catabolism, but an obvious infection or clear exacerbating factor is not always present. Because of the risk of multiorgan system failure, measurement of liver enzymes, blood urea nitrogen, creatinine, and creatine phosphokinase levels can also be helpful. Metabolic studies include measurement of lactate, ammonia, plasma amino acids, carnitine, acylcarnitine profile, and urine organic acids. Freezing aliquots of serum and urine and contacting a metabolic specialist for further guidance with respect to specialized testing are appropriate alternatives if your laboratory does not routinely handle such specimens.

Fatty acid oxidation disorders can progress to multiorgan system failure (Reye-like syndrome). Seizures, cardiomyopathy, cardiac arrest, liver failure, and kidney failure might supervene. There is a significant risk of death.

Secondary to therapeutic interventions, cerebral edema of unclear pathogenesis might occur in some IEMs, particularly those with hyperammonemia. This complication can also occur if a high rate of relatively dilute dextrose solution (eg, 5% dextrose) is used. Close monitoring of fluid resuscitation and neurologic status is therefore imperative.

Photo Credits Opener © Ted Kinsman/Photo Researchers, Inc.; 16-2 © Visuals Unlimited Unless otherwise indicated, all photographs and illustrations are under copyright of Jones & Bartlett Learning, courtesy of Maryland Institute for Emergency Medical Services Systems, or the American Academy of Pediatrics. Some images in this book feature models. These models do not necessarily endorse, represent, or participate in the activities represented in the images.

Chapter Review 16-35

Copyright © 2012 by the American Academy of Pediatrics and the American College of Emergency Physicians